SQE Property Practice Flashcards

You may prefer our related Brainscape-certified flashcards:
1
Q

A solicitor acts for a buyer of a registered property. The proprietorship register reveals that there are two registered proprietors and contains this entry:

“RESTRICTION: No disposition by a sole proprietor of the registered estate (except a trust corporation) under which capital money arises is to be registered unless authorised by an order of the court”.

The seller’s solicitors have confirmed that the two proprietors were married but that one of them is now deceased.

Which of the following best describes the position in which the buyer will obtain good legal and beneficial title?

A trustee needs to be appointed and only that trustee needs to execute the transfer deed.

Option b: Only the surviving proprietor needs to execute the transfer deed as the proprietors were married.

Option c: A second trustee needs to be appointed and both the second trustee and the surviving proprietor need to execute the transfer deed.

Option d: The surviving proprietor needs to execute the transfer deed and there needs to be an indemnity covenant added to the transfer deed.

Option e: The surviving proprietor needs to execute the transfer deed and an order of the court must be obtained.

A

Option C is correct. The entries in the proprietorship register confirm that the property was held as tenants in common. In the absence of an order of the court authorising the execution of the transfer deed by a sole trustee, a second trustee should be appointed in accordance with the Trustee Act 1925, so that two trustees are able to sign the transfer deed.

Option A is wrong. Whilst a second trustee will need appointing, as explained above, two trustees must sign the transfer deed. One person acting alone will not satisfy the restriction.

Option B is wrong. As the property was held as tenants in common, it is irrelevant whether the proprietors were married. A second trustee must be appointed in accordance with the Trustee Act 1925 so that two trustees are able to sign the transfer deed.

Option D is wrong. An indemnity covenant has no bearing on this issue.

Option E is wrong. It is the word “must” that makes this option wrong. Whilst is it possible to acquire a court order to authorise the sale by a single trustee, it is also possible to appoint a second trustee in accordance with the Trustee Act 1925 so that two trustees are able to sign the transfer deed.

How well did you know this?
1
Not at all
2
3
4
5
Perfectly
2
Q

A client is looking to acquire a property called Red Acres. The official copies for Red Acres include the following entry:

“The Property is subject to the rights granted in a conveyance of Blue Acres dated 7th June 1969 and made between John Smith (1) and Jennifer Jones (2)”

“A copy of this conveyance is filed at HM Land Registry “

Which of the following best describes the course of action the solicitor should pursue?

Option a. As the rights granted to Blue Acres will bind Red Acres enquiries should be made of the Seller and further details obtained from Land Registry.
selected

Option b: As the Seller will not have any information about these rights it is not worth raising requisitions in relation to them and further details should be obtained from Land Registry.

Option c: As the rights granted to Blue Acres do not bind Red Acres there will be no need to investigate further.

Option d: The rights referred to benefit Red Acres and so no further due diligence is required.

Option e: The rights granted to Blue Acres will only bind the client if he provides an indemnity covenant so advise the client not to give such a covenant.

A

Option A is the best answer. The rights granted to Blue Acres over Red Acres are referred to on the register and will bind Red Acres. Further information is therefore required of the Seller and a copy of the conveyance should be obtained from Land Registry. As a result, Option C is wrong.

Indemnity covenants are only relevant in the case of covenants (not rights) and so Option E is wrong.

Option D is wrong as the rights burden Red Acres; they do not benefit Red Acres.

The Seller can be asked questions on the rights and may have information on them. So, requisitions should be raised and Option B is therefore not the best answer.

How well did you know this?
1
Not at all
2
3
4
5
Perfectly
3
Q

A client is the purchaser of a small freehold commercial unit on a trading estate for £140,000. The solicitor has just completed the transaction and must now deal with Stamp Duty Land Tax (‘SDLT’).

What is the SDLT liability?

A. £300

B. £13,000

C. £26,000

D. Nil

E. £3,000

A

Option D is correct. SDLT is payable in bands and there are differing rates for residential and non-residential property. This property is non-residential, so up to £150,000 in value, the tax is nil.

Above £150,000 the tax is at a percentage depending on the value of the property. The total SDLT is therefore £Nil and Option D is the correct answer.

All other options show wrong figures.

How well did you know this?
1
Not at all
2
3
4
5
Perfectly
4
Q

A solicitor has been consulted by a client who has just purchased a freehold property. The property consists of a large house with two large detached garages at the bottom of the drive. The client would like to convert the two garages into holiday cottages for let but has discovered that one of the garages is being used by a neighbour to park his two classic cars. The neighbour has shown the client a written document, which was signed by the previous owner eight years ago. The document states that the neighbour has been granted an exclusive right to occupy the garage for a period of ten years at a rent of £200 per quarter, with no other monies paid upfront. There was no record of this document in the official copy of the register.

Does the agreement bind the client?

A. Yes, because there is an equitable lease, which did not need to be registered.

Option b: Yes, because there is a legal lease, which did not need to be registered.

Option c: No, because there is an equitable easement, which should have been registered.
.
Option d: No, because there is a legal lease, which should have been registered.

Option e: No, because there is a licence, which will not be binding on the client.

A

Option B is correct as this falls within the parol lease exception, s.54(2) LPA 25. It is not a fixed term legal lease (not created by deed as per s.52(1) LPA 25) nor fixed term equitable lease (not created in accordance with s.2 LP(MP)A 89). However, on the facts rent is paid quarterly so there is an implied quarterly periodic tenancy, which will fall within the parol lease exception, s.54(2) LPA 25. It is for less than three years (each period of the lease is 3 months), at best rent, without a fine (no upfront payment mentioned in the facts) and in immediate possession. This will be overriding under Schedule 3 Paragraph 1 LRA 02.

Option A is wrong as an equitable lease would need to comply with s.2 LP(MP)A 89 and be in writing, contain all the agreed terms and be signed by both parties

Option C is wrong as the document grants exclusive possession, which prevents there being an easement.

Option D is wrong, as per the discussion above there is a legal lease, but on the facts for a term of less than 7 years so cannot be registered as a registrable disposition.

Option E is wrong, as there is an implied periodic tenancy which falls within the parol lease exception, s.54(2) LPA 25, as above.

How well did you know this?
1
Not at all
2
3
4
5
Perfectly
5
Q

A solicitor is advising the buyer of a commercial property. The buyer has agreed in principle to purchase the property, provided the buyer can obtain planning permission to alter the property. The seller wants to incur as few solicitors’ fees as possible and therefore wants there be no exchange of contracts.

Which of the following statements provides the best advice for the buyer client?

A. Exchange of contracts is useful where, as here, the parties are agreed that conditions must be fulfilled before completion can take place.

Option b: Exchange of contracts is necessary because it fixes the completion date.

Option c: The parties should agree to simultaneous exchange of contracts and completion.

Option d: Exchange of contracts is not compulsory but would be useful here because it gives the buyer time between exchange of contracts and completion in order to make their final preparations.

Option e: Exchange of contracts is necessary because it is required by the Law Society Conveyancing Protocol.

A

Option A is the best advice because it makes the seller commit now to selling the property to the buyer client if, in the future, the buyer’s application for planning permission succeeds. If the buyer’s application is unsuccessful the buyer does not have to buy the property.

Option B is not the best advice because fixing the completion date is not the buyer’s concern here.

Option C is not the best advice because, whilst it is possible to simultaneously exchange contracts and complete, it would not help the buyer to obtain the position set out at A above.

Option D is not the best advice because having time to make final preparations is not the buyer’s concern here.

Option E is not the best advice because exchange of contracts is not required by the Law Society Conveyancing Protocol.

How well did you know this?
1
Not at all
2
3
4
5
Perfectly
6
Q

A seller of a freehold commercial property replies to enquiries before contract made by the buyer. The seller’s replies confirm that the property was built 10 years ago.

Which of these statements sets out the correct value added tax (VAT) position in respect of the sale of the property?

A. The sale of the property is a standard-rated supply and VAT will be charged at 20%.

Option b: The sale of the property is an exempt supply and no VAT can be charged.

Option c: The sale of the property is a zero-rated supply, however the seller can opt to charge VAT at the standard rate of 20%.

Option d: The sale of the property is an exempt supply however the seller can opt to charge VAT at the standard rate of 20%.

Option e: The sale of the property is a zero-rated supply and VAT is charged at 0%.

A

Option D is correct. Since the property is more than three years old, it is classified as an old commercial building for VAT purposes. The sale of an old commercial building is an exempt supply but the seller can decide to exercise its option to tax and charge VAT at the standard rate of 20%.

Option A is wrong because the sale of an old commercial building is an exempt supply, subject to the option to tax.

Option B is wrong because although the sale of an old commercial building is an exempt supply, the seller can waive that exemption and opt to tax the property.

Option C is wrong because the sale of an old commercial property is an exempt supply subject to the option to tax. Zero-rated supplies are taxable, but, as the name suggests, are subject to VAT at 0%. Exempt supplies are not subject to tax.

Option E is wrong because the sale of an old commercial building is an exempt supply, subject to the option to tax.

How well did you know this?
1
Not at all
2
3
4
5
Perfectly
7
Q

A buyer wishes to purchase a registered freehold property. The buyer’s solicitor is investigating the title and discovers that the Official Copies show the following entries.

Entry 2 of the Proprietorship Register:

“The transfer to the proprietor contains a covenant to observe and perform the covenants referred to in the Charges Register and of indemnity in respect thereof.”

Entry 3 of the Charges Register:

“A conveyance of the land dated 1 April 1928 made between (1) the vendor and (2) the purchaser contains the following covenant:

The purchaser with the intent and so as to bind (as far as practicable) the property hereby conveyed and to benefit and protect the Retained Land hereby covenants with the vendor that he and his successors in title will at all times hereafter not allow the fence on the northern boundary to fall in to disrepair.”

Will the buyer be obliged to maintain the fence as the new registered proprietor of the land?

Option A. Yes, because the covenant is registered then the buyer has notice and so will be bound by it.

Option b: No, because the indemnity is a personal covenant and as such will not bind the buyer.

Option c: No, because the burden of a positive covenant will never bind a buyer as it does not run to a successor in title.

Option d: Yes, because this is a registered restrictive covenant and so the burden runs with the land.

Option e: Yes, because the buyer will be required to give an indemnity covenant to the seller offering an indirect means of enforcing the covenant by way of a chain of indemnity.

A

Option E is correct. The covenant not to allow the fence to fall into disrepair is positive in nature as it requires expenditure i.e. to repair if it falls into disrepair. The burden of a positive covenant does not pass to a successor in title at law but there are indirect means of enforcement possible. This is an example of a chain of indemnities which work as a personal contractual agreement between a seller and buyer each time the property is sold. The chain of indemnities is already in existence, as evidenced by Entry 2 of the Proprietorship Register and the seller will require the buyer to continue this on, offering a means of enforcing the covenant indirectly against the buyer. As such, the buyer will be obliged to maintain the fence.

Option A is wrong as the statement only applies to restrictive covenants: the fact that a positive covenant appears on the Official Copies is insufficient to bind a successor.

Option B is wrong as whilst the covenant in the Proprietorship Register is personal, the seller will require the new buyer to enter into a new personal covenant, as discussed above.

Option C is wrong as there are indirect means of enforcing a positive covenant against a successor in title, as discussed above.

As the covenant not to allow the fence to fall into disrepair is positive, Option D is wrong.

How well did you know this?
1
Not at all
2
3
4
5
Perfectly
8
Q

A buyer is purchasing a residential property and is funding the purchase with the aid of a mortgage. The mortgage is on standard terms.

Will the buyer’s solicitor be able to act for both the buyer and the lender during the transaction?

A. Yes, because as the mortgage is on standard terms, it is possible to conclude that there is a ‘substantially common interest’ in that both the borrower and the lender want a good and marketable title to the property to be acquired.

Option b: Yes, because no conflict of interest can arise between buyer and lender. It is standard practice for the buyer’s solicitors to act for the lender on residential matters irrespective of the terms of the mortgage.

Option c: No, because the buyer and lender do not have a substantially common interest. Each party wants very different things in terms of the transaction: the buyer wants a property which has a good marketable title, the lender simply wants to ensure it can register its mortgage over the property.

Option d: No, because it is common for a conflict of interest to arise in such a transaction. Therefore, the lender will not want the buyer’s solicitors to act for it. It is normal practice for the lender to be separately represented.

Option e: No, because if the solicitor acts for both parties it will conflict with the solicitor’s own personal interest in the transaction and the solicitor should not therefore act for both.

A

Option A is correct. In residential transactions, the buyer will often be borrowing from a high street bank or building society where the mortgage is offered on standard terms and conditions with no negotiation being necessary. Therefore, it is likely that either no conflict will arise between the parties or that if there is a conflict, it will fall within the ‘substantially common interest exception’.

Option B is wrong. It is not standard practice. The solicitor will on each occasion assess the transaction based on the SRA Code of Conduct. There are occasions were a conflict can arise and the solicitor should not act for both parties (by way of example, where the terms of the mortgage offered are unfair to the buyer, the buyer is unable to comply with the mortgage terms or substantial negotiation will be required). On such occasions it would not be suitable for the buyer’s solicitor to act, irrespective of the fact that the matter relates to residential property.

Option C is wrong. Both the lender and the buyer will want a good and marketable title to the property. The lender is not solely concerned with registering the mortgage; the lender has an interest in the property and will want to ensure that the property has a suitable value and is marketable.

Option D is wrong. This is a transaction relating to a residential property and the mortgage is based on standard terms, therefore it is unlikely that the lender will need to be separately represented.

Option E is wrong. The fact that a solicitor acts for both parties will not mean that this will conflict with the solicitor’s own interest. There is nothing on the facts to suggest that the solicitor has a personal interest in the transaction.

How well did you know this?
1
Not at all
2
3
4
5
Perfectly
9
Q

A solicitor is contacted by two existing clients concerning the transfer for full market value of a property by one client to the other client. The heads of terms have already been agreed between the seller and the buyer. They include that the transfer must complete very quickly and that the buyer and the seller will instruct the solicitor to act for them both to speed up the process.

Can the solicitor accept instructions from both the seller and the buyer in these circumstances?

A. Yes, because the parties have already agreed heads of terms so there is no conflict of interest or significant risk of conflict of interest.

Option b: Yes, because a solicitor can act where the clients are competing for the same objective, in this case the quick transfer of the property.

Option c: No, because the while solicitors can on occasion act for two parties that have a substantial common interest, this does not apply to a property purchase.

Option d: No, because the speed with which the parties will be able to complete the transaction is not within the solicitor’s control.

Option e: Yes, because both the seller and the buyer are existing clients and therefore there is no possibility that they have unequal bargaining power.

A

Option C is correct. Solicitors can sometimes act for more than one client where the parties have a substantially common interest in relation to the matter. However, the Law Society has stated that “the SRA’s exception for circumstances where the two clients have a ‘substantially common interest’ does not apply to a property purchase. Although both clients will have a common interest in completing the sale, they also have different interests, since one is buying and one is selling.” The parties’ mutual desire to transfer the property or to do it quickly, does not extinguish the conflict of interest or significant risk of a conflict of interest. Thus applying paragraph 6.2 in the SRA Code of Conduct, the solicitor cannot act for both parties.

Option A is wrong because agreeing the heads of terms does not prevent the possibility of the parties arguing later, e.g. if something is revealed in the searches and enquiries that leads the buyer to seek a reduction in the purchase price.

Option B is wrong because the exception in paragraph 6.2(b) in the SRA Code of Conduct does not apply to property transactions in a buyer and seller situation.

Option D is wrong because the solicitor has not been asked to give an undertaking to speed up the completion of the transfer therefore the issue of whether or not the speed of the transaction is under the solicitor’s control is irrelevant.

Option E is wrong because whether or not the solicitor has acted for both parties in the past does not affect the parties’ bargaining position. The relevant point is as, the Law Society has stated, that “the SRA’s exception for circumstances where the two parties have a ‘substantially common interest’ does not apply to a property purchase”.

How well did you know this?
1
Not at all
2
3
4
5
Perfectly
10
Q

A client has agreed a sale of a freehold property it owns for £1,200,000.00, exclusive of value added tax (VAT), to a property investment company. The property was originally a cotton mill built in 1825 but was converted into offices by the client two years ago. The client informs its solicitor that it has incurred a significant amount of costs, including VAT, on carrying out the conversion work and wishes to recover as much of that VAT as possible on the sale. The solicitor is in the process of drafting the contract incorporating the Standard Commercial Property Conditions (3rd edition – 2018 Revision) (‘SCPC’) to send to the buyer’s solicitor for approval.

Which of the following statements best describes how the solicitor should deal with VAT in the draft contract?

A. The incorporation of the SCPC means that nothing further needs to be included in the draft contract to deal with VAT.

Option b: A special condition should be drafted to the effect that the purchase price of £1,200,000.00 is inclusive of VAT.

Option c: The solicitor should expressly incorporate the relevant optional standard condition in the SCPC relating to VAT.

Option d: A special condition should be drafted to disapply the SCPC insofar as they relate to VAT.

Option e: A special condition should be drafted to the effect that the sale is not a chargeable supply and that the seller will not exercise its option to charge VAT in any circumstances.

A

Option A is correct. This is a sale of an ‘old’ commercial freehold property and so the transaction is exempt from VAT, subject to the seller’s option to tax. Given the seller client’s instructions, the option to tax should be exercised in order that the VAT incurred by the seller on the conversion works can be offset against the VAT charged to the buyer on the sale. The effect of the SCPC being incorporated into the contract is that, by virtue of SCPC 2, the seller warrants that the sale is a chargeable supply and the buyer is to pay an additional amount equal to the VAT on completion. In this way, nothing further needs to be done to the draft contract to deal with VAT.

Option B is wrong because such a special condition would have the effect of overriding SCPC 2, with the effect that the seller would have to account for the VAT from the purchase price agreed instead of the buyer paying an amount equal to the VAT on top of the agreed purchase price.

Option C is wrong because incorporating the optional standard condition in Part 2 condition A1 by way of special condition would have the effect of disapplying SCPC 2 so that the sale would not constitute a chargeable supply for VAT and the buyer would not be required to pay VAT, unless there was a change in the law obliging the buyer to do so.

Option D is wrong because this would have the effect of the buyer being able to argue that the sale is not a chargeable supply and so no VAT would be payable by it.

Option E is wrong because it would have a similar effect to incorporating the relevant optional standard condition in Part 2 condition A1 and would prevent the seller from charging VAT on the sale in any circumstances.

How well did you know this?
1
Not at all
2
3
4
5
Perfectly
11
Q

On exchange of contracts for the purchase of a property a buyer has paid to the seller’s solicitors a deposit equal to 10% of the purchase price. Completion has not yet taken place.

Which one of the following statements regarding the deposit is correct?

A. If the deposit has been paid to the seller’s solicitor as agent for the seller, the seller’s solicitor will have to pay the deposit to the estate agents.

Option b: If the deposit has been paid to the seller’s solicitor as stakeholder, whether the contract incorporates the Standard Conditions or the Standard Commercial Conditions, the deposit can never be used by the seller until completion.

Option c: Whether the contract incorporates the Standard Conditions or the Standard Commercial Property Conditions, if the buyer fails to complete the purchase, the seller can rescind the contract and forfeit and keep the deposit.

Option d: Whether the contract incorporates the Standard Conditions or the Standard Commercial Property Conditions, provided the deposit is paid with funds from a clearing bank, that account can be in the name of the buyer or the buyer’s solicitor.

Option e: It is never possible for a buyer to pay less than 10% of the purchase price.

A

Option C is correct.as the Standard Conditions and the Standard Commercial Property Conditions both provide that the seller can rescind the contract and forfeit and keep the deposit if the buyer fails to complete.

Option A is wrong because if the deposit is paid to the seller’s solicitor as agent for the seller it means that the seller’s solicitor can pay the deposit to the seller immediately without waiting for completion.

Option B is wrong because the Standard Conditions provide that if the seller has agreed to buy another property in England and Wales as the seller’s residence, the seller may use all or part of the deposit as a deposit in that transaction.

Option D is wrong because the Standard Conditions and the Standard Commercial Property Conditions both provide that the funds to pay the deposit must come from an account at a clearing bank in the name of a conveyancer.

Option E is wrong because the seller and the buyer are free to agree the payment of a deposit of less than 10% of the purchase price.

How well did you know this?
1
Not at all
2
3
4
5
Perfectly
12
Q

A solicitor is asked to advise a client regarding the purchase of a registered commercial property. The client is purchasing the property with the assistance of a mortgage and the lender is separately represented.

The solicitor is asked to prepare a certificate of title.

Which of the following statements best describes the certificate of title which the solicitor will prepare?

A. The solicitor is likely to use the certificate of title in the form approved by the Law Society and UK Finance.

Option b: The final version of the certificate of title will be provided before exchange of contracts.

Option c: The certificate of title will not be disclosed to the lender as they are separately represented.

Option d: The solicitor cannot be sued by the lender if the information in the certificate of title is wrong as the solicitor does not act for the lender.

Option e: The certificate of title may be addressed to the client and its lender.

A

Option E is correct as a certificate of title will often be addressed to the buyer and the lender, sometimes just the lender.

Option A is wrong. On the facts this is a commercial transaction. In commercial transactions, the lender is likely to require a more detailed certificate of title, such as the one produced by the City of London Law Society. The form of certificate of title approved by the Law Society and UK Finance is used in residential transactions.

Option B is wrong because the final version of the certificate of title will be provided immediately prior to completion, not before exchange of contracts. Drafts will be provided to the buyer/lender’s advisors at earlier stages in the transaction so they will have early warning of any major issues.

Option C is wrong because even when a lender is separately represented, it is common for the buyer’s solicitor to report to the lender on the results of the title investigation and the pre-contract searches and enquiries. The certificate of title will provide such information.

Option D is wrong, if any of the information in the certificate is wrong, the lender can sue the firm which gave the certificate because there are warranties as to the correctness of the information contained within the certificate of title.

How well did you know this?
1
Not at all
2
3
4
5
Perfectly
13
Q

In which of the following circumstances should a solicitor acting on a sale and purchase of a commercial property recommend that in the contract the purchase price is expressed to be inclusive of VAT?

A. The solicitor is acting for a buyer which is an insurance company.
selected

Option b: The solicitor is acting for a buyer which is a firm of accountants.

Option c: The solicitor is acting for a seller and the property has been recently constructed.

Option d: The solicitor is acting for a seller and the property is a 1950s office building.

Option e: The solicitor is acting for a seller and the buyer is an insurance company.

A

Option A is correct. An insurance company is ‘VAT-sensitive’, ie it makes exempt supplies and may not be able to recover any VAT it has to pay on the purchase price. By contrast, a firm of accountants makes mainly standard rated supplies and is unlikely to object to paying VAT in addition to the purchase price since it will recover it (so option B is not the best answer).

A seller should never be advised by its own solicitor to make the purchase price inclusive of VAT (so options C, D and E are wrong). This is because if the supply is standard-rated (as with a new building), or it wants to opt to tax in order to recover VAT paid on refurbishment (as with the 1950s building), or the law changes the VAT status of the sale between exchange and completion, the seller will be unable to add the VAT to the agreed price. The sale proceeds will be reduced by the amount of VAT payable to HM Revenue and Customs.

Of course the seller may have to agree to a VAT inclusive purchase price as part of the commercial terms for the transaction, in which case, the seller’s solicitor must draw their client’s attention to the possible financial consequences of that decision.

How well did you know this?
1
Not at all
2
3
4
5
Perfectly
14
Q

A solicitor acts for a buyer who is buying a registered freehold property from the executor of a deceased owner. The official copies state that the class of title is absolute title. The executor is described in the contract as the seller. The solicitor is explaining to the client the concept of ‘limited title guarantee’ referred to in the contract.

Which of the following is the best advice to the buyer about the seller selling with limited title guarantee?

A. The seller should be selling the property with full title guarantee if the grant of probate appointing the executor has been granted, rather than limited title guarantee.

Option b: Limited title guarantee means that there is an implied covenant that the seller has not incumbered the title to the property and the seller is not aware that anyone else has done so since the last disposition for value.

Option c: The seller should not be selling the property with limited title guarantee because the class of the registered title is absolute title.

Option d: Limited title guarantee means that there is an implied covenant that the property is disposed free from incumbrances other than those the seller does not know about and could not reasonably know about.

Option e: Limited title guarantee means that once the transfer to the buyer has taken place, the class of title on the proprietorship register will be changed to qualified title.

A

Option B is the best answer as it is the correct definition of limited title guarantee.

Option A is not the best answer as you would expect an executor to sell with limited title guarantee because they will have limited knowledge of the property.

Option D describes full title guarantee. Options C and E are not the best answers as title guarantee for the purpose of the contract does not link to the class of title in registered land; the property may continue to be registered with absolute title even though it has been sold to the buyer by a seller only offering limited or no title guarantee. Title guarantee is about the seller; class of title is about the property.

How well did you know this?
1
Not at all
2
3
4
5
Perfectly
15
Q

A solicitor has exchanged contracts to buy a commercial property for a client. Title to the property is registered at Land Registry. The contract incorporates the Standard Commercial Property Conditions (Third Edition-2018 Revision) without amendment. The client wants to know if he now has an interest in the property and what happens if the property should be damaged by a bad storm that is forecast before completion.

What advice should be given on whether the client has an interest in the property and who bears the risk of storm damage?

A. From exchange of contracts, the client has a legal interest in the property and bears the risk of any storm damage to the property.

Option b: From exchange of contracts, the client has a beneficial interest in the property and bears the risk of any storm damage to the property.

Option c: From exchange of contracts, the client has neither a legal or beneficial interest in the property and does not bear the risk of any storm damage to the property.

Option d: From exchange of contracts, the client has a legal interest in the property, but the seller retains the risk of any storm damage to the property.

Option e: From exchange of contracts, the client has a beneficial interest in the property, but the seller retains the risk of any storm damage to the property.

A

Option B is the correct answer. On exchange, the beneficial (or equitable) interest in the property passes to the buyer, but the legal title remains with the seller. Under the Standard Commercial Property Conditions (Third Edition-2018 Revision) risk in the property passes on exchange and so if the property is damaged by a storm after exchange, the buyer will bear the risk of that damage (and must still buy the property).

Option A is wrong. The buyer does not acquire the legal interest in the property until completion has taken place and the transfer to the buyer has been registered at Land Registry.

Option C is wrong. The buyer does have a beneficial interest from exchange and the risk of damage passes to the buyer.

Option D is wrong. The buyer only has an equitable interest but still bears the risk of damage from exchange.

Option E is wrong.it is correct that the buyer has an equitable interest but wrong to say the seller retains the risk of damage, as this risk passes to the buyer on exchange.

How well did you know this?
1
Not at all
2
3
4
5
Perfectly
16
Q

A solicitor is asked to advise a client regarding the purchase of a registered freehold property and is reviewing the draft contract. The solicitor is explaining to the client the concept of title guarantee referred to in the contract.

Which of the following statements is correct?

A. Limited title guarantee means that the class of title in the Proprietorship register will be Absolute Title.

Option b: Full title guarantee means that there is an implied covenant that the land is disposed of free from incumbrances other than those the seller does not know about and could not reasonably know about.

Option c: Full title guarantee means that the class of title in the Proprietorship register will be Qualified Title.

Option d: Limited title guarantee means that there is an implied covenant that the land is disposed of free from incumbrances other than those the seller does not know about and could not reasonably know about.

Option e: An executor of a deceased estate selling a property will normally give full title guarantee.

A

Option B is correct as it describes full title guarantee. A full title guarantee implies more comprehensive implied covenants for title than would be the case with limited title guarantee including that the land is disposed free from incumbrances other than those the seller does not know about and could not reasonably know about.

Options A and C are wrong because title guarantee is not the same as class of title on the Register or linked to it. Title guarantee is about the seller, class of title is about the property.

Option D is wrong as this wording describes full, not limited, title guarantee.

Option E is wrong as executors will not normally give full title guarantee as they will likely have limited knowledge of the property.

How well did you know this?
1
Not at all
2
3
4
5
Perfectly
17
Q

Contracts have been exchanged this morning for the sale and purchase of an office building. The contract incorporates the Standard Commercial Property Conditions (Third Edition) (SCPCs) without amendment. Completion is to take place in two weeks. The seller has now informed its solicitor that it needs part of the deposit monies straight away to put down as a deposit for the purchase of another commercial building.

Which of the following statements best describes what the seller’s solicitor should do?

A. Explain to the client that the solicitor must retain the deposit until completion.
selected

Option b: Transfer the entire deposit to the seller immediately.

Option c: Explain to the client that the solicitor must retain the deposit until the transfer to the buyer has been registered.

Option d: Transfer to the seller only that part of the deposit required for the purchase of the new commercial building.

Option e: Serve the buyer with notice to complete so that if the buyer does not complete by the time stated in the notice the seller can use the deposit monies.

A

Option A is the best answer. The SCPCs provide for the deposit to be held by the seller’s solicitor as stakeholder. This means that once received from the buyer, the deposit cannot be released to the seller until completion.

Option B is wrong: the deposit would need to be held as agent for this to be correct.

Option C is not the best answer, as the deposit can be released at completion.

Option D is wrong: only when the Standard Conditions of Sale apply, is there provision for the property to be used on a related purchase.

Option E is wrong: a notice to complete cannot be served until completion is delayed beyond the contractual completion date.

How well did you know this?
1
Not at all
2
3
4
5
Perfectly
18
Q

A solicitor has been contacted by a prospective buyer of a residential property. The buyer tells the solicitor that he has viewed a property and made a verbal offer to the estate agent to purchase it for £350,000. The offer has been accepted by the seller. The parties have discussed which fittings the seller will leave for the buyer and when they wish to complete.

Does the buyer have a binding contract with the seller to purchase the property?

A. Yes, because consideration has been provided.

Option b: Yes, because there has been an offer and acceptance.

Option c: No, because a contract for the sale of land must be in writing and signed by both parties.

Option d: No, because a contract for the sale of land must be in the form of a deed.

Option e: No, because a contract for the sale of land must include a standard set of conditions.

A

Option C is the correct answer. The main requirements for creating a binding contract for the sale of land are set out in s 2(1) of the Law of Property (Miscellaneous Provisions) Act 1989. These are that the contract must be in writing; it must incorporate all the agreed terms; these must be contained in one document, or where contracts are exchanged, in each copy of the contract; and, finally, the contract must be signed by the parties. The agreement described in the question is verbal and as a result unsigned. It does not therefore satisfy the above requirements.

Option A is wrong. There is no consideration on the facts and in any event a contract for the sale of land must satisfy the additional requirements of s 2(1) of the Law of Property (Miscellaneous Provisions) Act 1989 (described above).

Option B is wrong. Offer and acceptance are requirements for a contract, but there are additional statutory requirements for a contract for the sale of land, as described above.

Option D is wrong. A legal estate can only be transferred by deed, (which is why a TR1 is a deed) but the contract does not have to be by deed.

Option E is wrong. Most contracts do include a set of standard conditions, but this is not a legal requirement.

How well did you know this?
1
Not at all
2
3
4
5
Perfectly
19
Q

A solicitor has just exchanged contracts on the purchase of a commercial property for a client. The contract incorporates the Standard Commercial Property Conditions (Third Edition-2018 Revision) without amendment. The exchange of contracts was carried out over the telephone using Law Society formula B.

Which of the following statements best describes steps that should be taken on the day of exchange by the buyer’s solicitor?

A. The buyer’s solicitor should send his client’s part of the contract and the deposit to the seller’s solicitor and advise the buyer to immediately insure the property.

Option b: The buyer’s solicitor should send his client’s part of the contract and the deposit to the seller’s solicitor and check the seller has insurance in place for the property.

Option c: The buyer’s solicitor should send his client’s part of the contract and the deposit to the seller’s solicitor and advise the buyer to insure the property from the completion date.

Option d: The buyer’s solicitor should send his client’s part of the contract to the seller’s solicitor and ask the buyer to send the deposit to the seller and to immediately insure the property.

Option e: The buyer’s solicitor should send his client’s part of the contract and the deposit to the seller and advise the buyer to immediately insure the property.

A

Option A is the best answer. By using Law Society formula B, the buyer’s solicitor has undertaken to send the deposit and contract to the seller’s solicitor. As the contract contains the Standard Commercial Property Conditions (Third Edition-2018 Revision), the risk in the property passes to the buyer on exchange and therefore the buyer must insure the property from exchange.

Option B is not the best answer. Whilst the seller may well retain its insurance over the property, the risk in the property passes to the buyer on exchange and therefore the buyer must insure the property from exchange.

Option C is not the best answer. The risk in the property passes to the buyer on exchange and therefore the buyer must insure the property from exchange, not from completion.

Option D is not the best answer. By using Law Society formula B, the buyer’s solicitor has undertaken to send the deposit and contract to the seller’s solicitor. The buyer’s solicitor should not rely on the buyer to send the deposit monies, as if the buyer does not do so the solicitor will be in breach of undertaking.

Option E is not the best answer. To comply with the Law Society Formula B undertakings the contract and deposit must be sent to the seller’s solicitor, not the seller.

How well did you know this?
1
Not at all
2
3
4
5
Perfectly
20
Q

A solicitor is acting for the seller of a freehold commercial property which was constructed a year ago. The buyer is a firm of accountants. The parties are nearly ready to exchange with completion planned for three weeks’ time.

Should the solicitor recommend to the seller that in the contract the agreed purchase price is expressed to be inclusive of VAT?

A. Yes, because it will be compulsory for VAT to be chargeable at the sale.

Option b: No, because making the purchase price inclusive of VAT will cause the buyer to try and reduce the purchase price.

Option c: Yes, because VAT will not be chargeable unless the seller opts to tax the property.

Option d: No, because this will reduce the amount of the purchase price available for the seller.

Option e: Yes, because VAT will only be chargeable if the government changes the VAT status of the transaction in the next three weeks.

A

Option D is correct. The seller should not be advised by its own solicitor to make the purchase price inclusive of VAT (so options A, C and E are wrong). This is because the supply of a ‘new’ building within three years of construction is standard-rated so VAT will have to be charged. With a VAT- inclusive purchase price, the seller will be unable to add the VAT to the agreed price so the sale proceeds will be reduced by the amount of VAT payable to HM Revenue and Customs.

Option B is wrong because an accountancy firm is not ‘VAT-sensitive’ (ie an entity which makes exempt supplies and may not be able to recover any VAT it has to pay on the purchase price). A firm of accountants makes mainly standard rated supplies and is unlikely to object to paying VAT in addition to the purchase price since it will recover it.

How well did you know this?
1
Not at all
2
3
4
5
Perfectly
20
Q

A solicitor is acting for the buyer of a property and is preparing to exchange contracts. The property is being sold for £500,000 and the contract incorporates the Standard Commercial Property Conditions (Third Edition-2018 Revision) with a special condition stating that any monies due under the contract can be paid either electronically or by client account cheque. The solicitor has agreed with the seller’s solicitor they will exchange using Law Society formula B. Completion will be 14 days after exchange.

Which of the following best describes what monies the buyer’s solicitor should obtain from the client to exchange?

A. The solicitor should obtain £500,000 in cleared funds to exchange, as the solicitor will be undertaking that all monies due under the contract can be paid.

Option b: The solicitor does not need to obtain any monies to exchange, as no monies are due until completion

Option c: The solicitor should obtain £50,000 in cleared funds to exchange, as the solicitor will be undertaking to send a 10% deposit on exchange.

Option d: The solicitor should obtain £25,000 in cleared funds to exchange, as the solicitor will be undertaking to send a 5% deposit on exchange.

Option e: The solicitor does not need to obtain any monies to exchange, as the client will send the deposit monies directly to the seller.

A

Option C is the best answer. The Standard Commercial Property Conditions (Third Edition-2018 Revision) (‘SCPC’) provide for a 10% deposit to be paid by the buyer on exchange. When the buyer’s solicitor exchanges using Law Society formula B, the buyer’s solicitor will undertake to send the deposit monies to the seller’s solicitor on the day of exchange. According to the special condition referred to in the facts, these can be sent electronically or by client account cheque. The buyer’s solicitor should therefore have cleared funds to ensure he can comply with the undertaking and the terms of the contract.

Option A is not the best answer. Whilst the buyer’s solicitor could collect the whole of the purchase monies in advance of exchange, the buyer’s solicitor is only undertaking to send the deposit on the day of exchange.

Option B is wrong- the deposit must be paid on exchange.

Option D is wrong. The SCPC provide for a 10% deposit. The facts do not indicate a special condition has been agreed to provide for a 5% deposit.

Option E is wrong. Formula B requires the buyer’s solicitor to send the deposit to the seller’s solicitor.

How well did you know this?
1
Not at all
2
3
4
5
Perfectly
21
Q

A solicitor is advising the buyer of a commercial property. The contract for the sale of the property has been drafted by the seller’s solicitor and sent to the buyer’s solicitor for review. The draft contract incorporates the Standard Commercial Property Conditions (Third Edition – 2018 Revision) (‘SCPC’).

The section on the front page of the pre-printed form of contract concerning the contract rate has been left blank. There are no other relevant special conditions.

Will compensation be payable under the contract should completion be delayed due to buyer default?

A. Yes, because the SCPC contain a contractual entitlement to compensation, the buyer will pay compensation to the seller, calculated at The Law Society’s interest rate from time to time in force.
selected

Option b: No, because the section concerning the contract rate has been left blank, no compensation will be payable in the event of late completion.

Option c: Yes, because the SCPC contain a contractual entitlement to compensation, the buyer will pay compensation to the seller, calculated at 4% above the Law Society’s interest rate from time to time in force.

Option d: No, because only the seller has an obligation to pay compensation for late completion under the SCPC.

Option e: No, because late completion only entitles the seller to claim damages where loss has been.

A

Option A is correct. The forms of contract purchased from law stationers contain a space on the front page in which the contract rate may be inserted. If it is wished to rely on Condition 1.1.1(e) this can be left blank. If left blank, compensation will be payable to the seller at the contract rate specified in Condition 1.1.1(e) in the event that the buyer defaults in performing its obligations under the contract and completion is delayed. This is confirmed at paragraphs 10.3.1 and 10.3.2 SCPC. The contract rate specified in Condition 1.1.1(e) is “The Law Society’s interest rate from time to time in force.”

Option B is wrong because in that instance the contract rate set out in Condition 1.1.1(e) can be relied upon.

Option C is wrong because the contract rate specified in Condition 1.1.1(e) is “The Law Society’s interest rate from time to time in force.” This is set at 4% above the base lending rate of Barclays Bank plc.

Option D is wrong because it is only the buyer that has an obligation to pay compensation for late completion under paragraph 10.3.1 SCPC.

Option E is wrong because SCPC 10.3 requires payment of compensation for delayed completion irrespective of whether the seller has suffered any loss.

How well did you know this?
1
Not at all
2
3
4
5
Perfectly
22
Q

A solicitor is advising the buyer of a commercial property. The contract for the sale of the property has been drafted by the seller’s solicitor and sent to the buyer’s solicitor for review. The draft contract incorporates the Standard Commercial Property Conditions (Third Edition – 2018 Revision) (‘SCPC’).

The section on the front page of the pre-printed form of contract concerning the contract rate has been left blank. There are no other relevant special conditions.

Will compensation be payable under the contract should completion be delayed due to buyer default?

A. Yes, because the SCPC contain a contractual entitlement to compensation, the buyer will pay compensation to the seller, calculated at The Law Society’s interest rate from time to time in force.

Option b: No, because the section concerning the contract rate has been left blank, no compensation will be payable in the event of late completion.

Option c: Yes, because the SCPC contain a contractual entitlement to compensation, the buyer will pay compensation to the seller, calculated at 4% above the Law Society’s interest rate from time to time in force.

Option d: No, because only the seller has an obligation to pay compensation for late completion under the SCPC.

Option e: No, because late completion only entitles the seller to claim damages where loss has been suffered.

A

Option A is correct. The forms of contract purchased from law stationers contain a space on the front page in which the contract rate may be inserted. If it is wished to rely on Condition 1.1.1(e) this can be left blank. If left blank, compensation will be payable to the seller at the contract rate specified in Condition 1.1.1(e) in the event that the buyer defaults in performing its obligations under the contract and completion is delayed. This is confirmed at paragraphs 10.3.1 and 10.3.2 SCPC. The contract rate specified in Condition 1.1.1(e) is “The Law Society’s interest rate from time to time in force.”

Option B is wrong because in that instance the contract rate set out in Condition 1.1.1(e) can be relied upon.

Option C is wrong because the contract rate specified in Condition 1.1.1(e) is “The Law Society’s interest rate from time to time in force.” This is set at 4% above the base lending rate of Barclays Bank plc.

Option D is wrong because it is only the buyer that has an obligation to pay compensation for late completion under paragraph 10.3.1 SCPC.

Option E is wrong because SCPC 10.3 requires payment of compensation for delayed completion irrespective of whether the seller has suffered any loss.

How well did you know this?
1
Not at all
2
3
4
5
Perfectly
23
Q

A buyer is purchasing a commercial property and is about to exchange contracts. The purchase price is £250,000. The contract incorporates the Standard Commercial Property Conditions (Third Edition-2018 Revision) without amendment.

What deposit will be payable on exchange and what can the seller’s solicitor do with the deposit?

A. £25,000 will be payable on exchange and the seller’s solicitor can hand over the deposit to the seller upon receipt.

Option b: £12,500 will be payable on exchange and the seller’s solicitor can hand over the deposit to the seller upon receipt.

Option c: £12,500 will be payable on exchange and the seller’s solicitor cannot hand it over to the seller until completion.

Option d: £25,000 will be payable on exchange and can be used by the seller’s solicitor on a related purchase.

Option e: £25,000 will be payable on exchange and the seller’s solicitor cannot hand it over to the seller until completion.

A

Option E is the correct answer. The Standard Commercial Property Conditions (Third Edition-2018 Revision) (‘SCPC’) provide for a 10% deposit to be paid which is to be held as stakeholder. Where a deposit is held as stakeholder, it cannot be handed over to the seller until completion.

Option A is wrong. This describes the position where the deposit is held as agent. The contract provides for the deposit to be held as stakeholder.

Option B is wrong. This describes a 5% deposit held as agent. The contract provides for a 10% deposit to be held as stakeholder.

Option C is wrong. This describes a 5% deposit. The contract provides for a 10% deposit.

Option D is wrong. The deposit can be used on a related purchase under the Standard Conditions, but not under the SCPC.

How well did you know this?
1
Not at all
2
3
4
5
Perfectly
24
Q

A solicitor is acting for a client who intends to purchase a property. The client inspected the property and noted that it is located near a medieval church which is used by the local people. Contracts have not been exchanged, but pre-contract enquiries of the seller have been raised. The current seller has owned the property since June 2010 and the property is registered at Land Registry. The solicitor wants to establish if the client may have any liabilities in respect of the church when it buys the property.

Which of the following statements best describes the additional searches, enquiries and investigations the solicitor should carry out to establish this?

A. The solicitor should carry out a Chancel Repair Screening search and a Local Land Charges (LLC1) search.
selected

Option b: The solicitor should carry out a Chancel Repair Screening search and an Index Map search.

Option c: The solicitor should carry out a Chancel Repair Screening search and review the official copies of the Seller’s title.

Option d: The solicitor should carry out a Local Land Charges (LLC1) search and review the official copies of the Seller’s title.

Option e: The solicitor should raise both standard and optional enquiries with the Local Authority (CON29 and CON29O).

A

The solicitor should carry out the Chancel Repair Screening search. For historical reasons certain properties in certain parishes could be under an obligation to pay the cost to repair the chancel of the parish church. Since 13th October 2013, chancel repair liability is no longer an overriding interest. Chancel repair liabilities can now only be protected by notice on the title of the burdened property. This registration can take place at any time until a transfer for value occurs on or after 13th October 2013.The current owner has owned the property since June 2010. No transfer for value has taken place since 13th October 2013. This therefore means potentially the liability can still be registered at any time on the title. This means the solicitor must carry out the Chancel Repair Screening search to ascertain whether the property is within a liable parish.

The solicitor should also request copies of the title from the Land Registry to check whether the chancel repair liability has already been registered on the title. Option C is therefore the best answer.

Options A and D are not the best answers as they both refer to the Local Land Charges search (LLC1). Whilst this search is routinely carried out, the LLC1 search result only discloses financial charges or restrictions on land that have been imposed by public authorities under statute and will not reveal any chancel repair liability.

Option B is wrong, as an Index Map search would only reveal if the land or any part of it has been registered at Land Registry. On the facts we are told that the land is registered at the Land Registry and an index map search will not reveal information on chancel repair liability.

Option E is wrong. Enquiries of the Local Authority (CON29) reveal an enormous quantity of non-statutory information which affects a property but will not disclose any chancel repair liability.

How well did you know this?
1
Not at all
2
3
4
5
Perfectly
25
Q

A solicitor is acting for the buyer on the purchase of a registered freehold property which is in the pre-contract stage. The solicitor is also acting for the buyer’s lender. The buyer is an individual and the seller is a company. The solicitor’s firm’s policy is to carry out solvency checks where necessary at the pre-contract stage.

What searches should the solicitor carry out to check the solvency of the parties?

A. A company search against the seller and a K16 bankruptcy search against the buyer.

Option b: A company search against the buyer and a K16 bankruptcy search against the seller.

Option c: A company search against the buyer and the seller.

Option d: A K16 bankruptcy search against the buyer but no search against the seller.

Option e: A company search against the seller and the lender and a K16 bankruptcy search against the buyer.

A

Option A is the correct answer. As the seller is a company it is necessary to carry out a company search to check various matters including solvency. As the buyer is an individual it is necessary to carry out a k16 search at the Land Charges department to check for insolvency. All firms should carry out the K16 search as a pre-completion search (if acting for a lender), but on the facts of the question this firm also checks for solvency pre-contract.

Option B is wrong. As the seller is a company checks for insolvency must be made via a company search. As the buyer is an individual, searches for bankruptcy will be made using a K16 search at the Land Charges department.

Option C is wrong as a company search cannot be carried out against the buyer as they are an individual.

Option D is wrong. Although a K16 search against the buyer is appropriate, the solicitor will also need to carry out a company search against the seller to check various matters including solvency.

Option E is wrong as although the searches against the buyer and seller are correct, there is no need to search against the lender.

26
Q

A solicitor is acting for the seller on the sale of freehold property with an unregistered title. The solicitor is preparing for deduction of title to the property to the buyer’s solicitor and has examined the deeds and documents relating to the property.

Which of the following documents is the best candidate for a good root of title?

A. A Deed of Gift dated 1 October 1990.

Option b: A Conveyance of the property dated 13 June 1998.

Option c: A Mortgage of the property dated 13 June 1998.

Option d: A Grant of Probate dated 6 February 2010.

Option e: An Assent of the property dated 25 March 2010.

A

Option B is correct.

Under s 44 of the Law of Property Act 1925, a root of title must be at least 15 years old. The Deed of Gift in Option A is old enough and will probably fulfil the other criteria set out in s 44, but as it was not a transaction between third parties for valuable consideration, it will not offer the double guarantee like the Conveyance and the Mortgage (ie that title has been investigated for at least 30 years).

Both the Conveyance and the Mortgage are capable of being good roots of title and offer the double guarantee, but the Conveyance is preferable to the Mortgage as it is likely to contain a more detailed description of the property by reference to a plan and details of the incumbrances that burden the property and deal expressly with the legal and equitable interest in the property. Therefore, option C is not the best answer. The Grant of Probate and the Assent (options D and E) are too recent to be good roots of title.

27
Q

A solicitor acts for a client who is buying a residential registered freehold property.

Which one of the following statements is correct concerning the need to obtain pre-contract searches and enquiries?

A. The buyer’s solicitor should raise pre-contract searches on behalf of the buyer to comply with the principle of caveat emptor.
selected

Option b: The buyer’s solicitor should raise pre-contract searches and will expect the seller to disclose details of physical defects with the property.

Option c: The seller’s solicitor should raise pre-contract searches and forward the results to the buyer’s solicitors.

Option d: The buyer’s solicitor should raise pre-contract searches including a Land Charges Department search.

Option e: The buyer’s solicitor will only raise searches where indicated as necessary out of the results of the title investigation.

A

Option A is correct because searches and enquiries are raised on behalf of the buyer as a consequence of the principle of “buyer beware.”

Option B is wrong because the seller is under no obligation to disclose details of physical defects to the buyer.

Option C is wrong because since the risk of buying the property subject to undiscovered defects rests with the buyer, it is up to the buyer to make the necessary searches and check the results are satisfactory.

Option D is wrong because a Land Charges Department search is required where the land is unregistered and the facts tell us that the property being purchased by the buyer is registered.

Option E is wrong because searches and enquiries should be obtained and considered even where there are no problems arising from investigation of title.

28
Q

A solicitor acts for a buyer purchasing a factory to be converted into a car showroom (‘the change of use’). Factories and car showrooms fall within different use classes for the purposes of planning legislation.

The client will be installing large glass showroom windows to face the highway (‘the window installation’). At present there are no windows facing the highway.

Neither the change of use or window installation are covered by the Town and Country Planning (General Permitted Development) Order 2015 (‘GPDO’).

Which of the following best describes the position on planning permission?

A. The client will not need to apply for planning permission for the change of use but will require planning permission for the window installation.

Option b: The client will not need to apply for planning permission for the window installation but will require planning permission for the change of use.

Option c: The client will require planning permission for both the change of use and the window installation.

Option d: The client will not require planning permission for either the change of use or the window installation.

Option e: Planning permission is required but is automatically granted by the GPDO for both the change of use and the window installation.

A

Option C is best answer: Planning permission will be required for the change of use from a factory to a car show room, as a material change of use constitutes “development” for the purposes of S55 Town and Country Planning Act 1990. It will also be required for the window installation as building works materially affecting the external appearance of a property also constitute ‘development’.

Option A and B are therefore wrong: both suggest planning permission is only required for one aspect of the development. Option D is also wrong, as planning permission is required for the reasons stated above.

Option E is wrong as the facts indicate that the change of use and window installation do not amount to permitted development under the GPDO. As a result, an express planning application must be made.

29
Q

A solicitor is acting for a client who intends to purchase a property adjoining a canal. The client wants to know who is responsible to maintain the towpath and who has maintained it in the past. The seller has owned the property for about 10 years.

Which of the following statements best describes the pre-exchange searches and enquiries the solicitor should carry out to address this specific issue?

A. The solicitor should carry out a water and drainage search and raise specific pre-contract enquiries of the seller.

Option b: The solicitor should carry out a desktop environmental search and a flood search.

Option c: The solicitor should carry out a local land charges search (LLC1) and raise enquiries with the Canal & River Trust.

Option d: The solicitor should raise enquiries with the Canal & River Trust and raise specific pre-contract enquiries of the seller.

Option e: The solicitor should raise enquiries with the Canal & River Trust and carry out a flood search.

A

Option D is correct.

The client is concerned about the maintenance of the towpath of the canal. The solicitor must therefore raise enquiries with the Canal & River Trust who can provide specific information regarding liability for maintenance of the towpath. In addition, the solicitor should also raise specific pre-contract enquiries of the seller to find out further information regarding the maintenance regime in place and the level of financial contribution incurred.

The other searches will not address the client’s specific concern. The environmental search will only cover contamination risk and the flood search will only cover flood risk; accordingly options B and E are wrong The water and drainage search will cover whether the property is connected to the public water supply and the use of a public sewer. Option C is wrong as the local land charges search (LLC1) only discloses financial charges or restrictions on land that have been imposed by public authorities under statute, none of which apply to matters concerning the maintenance of canal towpaths.

30
Q

A solicitor acts for the buyer in a conveyancing transaction. In the replies to the pre-contract enquiries of the seller, the seller answers in reply as to the physical state of the property “Please rely on your own survey.”

Which ONE of the following statements BEST describes the position?

A. The buyer’s solicitor should request that the seller provides a fuller answer.

Option b: The seller is entitled to answer in that way but will have to indemnify the buyer if there are later revealed to be physical defects in the property.

Option c: The seller is entitled to answer in that way.

Option d: The seller must provide a fuller answer and will have to indemnify the buyer if there are later revealed to be physical defects in the property.

Option e: The buyer’s solicitor should request that the seller provides a fuller answer where the buyer has not instructed a surveyor to inspect the property.

A

C is the best answer. The seller does not have a duty to disclose physical defects in the property. This is why A is wrong. The seller has not provided a misleading reply to this specific enquiry. The seller will not be liable to indemnify the buyer in this respect. For these reasons, B, D and E are wrong.

31
Q

A solicitor is acting for the buyer of a freehold property. Investigation of title has not revealed any restrictive covenants related to use of the property. The pre-contract searches and enquiries have revealed that the designated planning use for the property is use as a grocery store within use class E of the Town and Country Planning (Use Classes) Order 1987. Following completion, the buyer wishes to open a fast food takeaway at the property, which is a sui generis use. Does the buyer need to apply for express planning permission in order to be able to use the property as a fast food takeaway?

A. Yes, because the proposal constitutes development and will not be covered by the GPDO.

Option b: No, because whilst the proposal constitutes a material change of use, planning permission is automatically granted under the Town and Country Planning (General Permitted Development) Order 2015 (‘GPDO’).

Option c: No, because there is nothing on the title restricting the use of the property.

Option d: No, because a change of use never constitutes development.

Option e: Yes, because any change of use requires express planning permission.

A

Option A is correct.

Planning permission is required for any activity which constitutes ‘development.’ Under s 55 of the Town and Country Planning Act 1990, ‘development’ includes the making of any material change in the use of any buildings or other land. Option D is wrong because a ‘material’ change of use will constitute development. A change between a use within use class E and a sui generis use is a material change of use.

Express planning permission is therefore required, unless planning permission is automatically granted under the GPDO (so Option E is wrong). However, this change of use will not be covered by the GPDO as it is a change of use between use class E and a sui generis use; therefore planning permission is not automatically granted and express planning permission will be required, so option B is wrong.

Planning permission is a separate issue to restrictive covenants on title, so option C is also wrong.

32
Q

A solicitor acting for a buyer of a commercial property has undertaken a Local Land Charges Search (LLC1). The property was built eight years ago, is surrounded by trees and fronts on to Boswell Road.

What might the results of this search reveal about the property?

A. Tree preservation orders and any planning permissions granted in respect of the property.

Option b: Tree preservation orders and confirmation as to whether the roads adjoining the property (including Boswell Road) are maintained by the local authority.

Option c: The existence of any enforcement notices served in respect of breach of planning control.

Option d: Information about proposed road schemes within a certain distance of the property.

Option e: Tree preservation orders and confirmation as to whether the drains and sewers serving the property are maintained by the local authority.

A

Option A is correct because these are matters that would be shown on an LLC1.

Option B is wrong because the adoption and maintenance of roads by the local authority would be revealed in the results to the Enquiries of the Local Authority (CON29) not the LLC1.

Option C is wrong because any enforcement notices in relation to planning would be revealed on the CON29 not the LLC1.

Option D is wrong because proposed road schemes would be revealed on the CON29 not the LLC1.

Option E is wrong because the maintenance of drains and sewers by the local authority would be shown on a water and drainage search.

33
Q

A solicitor is acting for the buyer of a freehold property. The pre-contract searches and enquiries have revealed that the designated planning use for the property is use as offices within use class E of the Town and Country Planning (Use Classes) Order 1987. Following completion, the buyer wishes to open a wine bar at the property, which is a sui generis use. Investigation of title has not revealed any restrictive covenants related to use of the property.

Does the buyer need to apply for express planning permission in order to be able to use the property as a wine bar?

A. No, because there is nothing on the title restricting the use of the property.

Option b: No, because a change of use never constitutes development.

Option c: No, because whilst the proposal constitutes a material change of use, planning permission is automatically granted under the Town and Country Planning (General Permitted Development) Order 2015 (‘GPDO’).

Option d: Yes, because any change of use requires express planning permission.

Option e: Yes, because the proposal constitutes development and will not be covered by the GPDO.

A

Option E is correct.

Planning permission is a separate issue to restrictive covenants on title, so option A is wrong.

Planning permission is required for any activity which constitutes ‘development.’ Under s 55 of the Town and Country Planning Act 1990, ‘development’ includes the making of any material change in the use of any buildings or other land. A change between a use within use class E and a sui generis use is a material change of use, so option B is not the best answer.

Express planning permission is therefore required, unless planning permission is automatically granted under the GPDO. This change of use will not be covered by the GPDO as it is a change of use between use class E and a sui generis use; therefore planning permission is not automatically granted and express planning permission will be required, so option C is not the best answer.

Option D is not the best answer because a ‘material’ change of use will constitute development.

34
Q

A solicitor is investigating title to an unregistered freehold property.

The epitome of title consists of copies of the following documents:

A conveyance dated 3 March 1975 between Mavis Wright (1) and Reginald Forster (2)
A legal mortgage dated 2 August 1980 between Reginald Forster (1) and The Royal Bank of Wales PLC (2)
Death certificate in the estate of Reginald Forster dated 12 October 2020
Grant of probate in the estate of Reginald Forster in favour of Stephen Forster and Heather Willis dated 20 December 2020
A clear land charges search against Reginald Forster for the period from 1975 to 2020
The solicitor has correctly identified that the 1975 conveyance is a good root of title.

Which of the following correctly lists the additional land charges searches the solicitor will have to undertake?

A. None, all necessary land charges searches have been undertaken.

Option b: Mavis Wright, 1926 – 1975 The Royal Bank of Wales PLC, 1980 - to present year Stephen Forster and Heather Willis, 2020 - to present year

Option c: Mavis Wright, 1926 – 1975 Reginald Forster, 1975 - 2020Stephen Forster and Heather Willis, 2020 - to present year

Option d: Stephen Forster and Heather Willis, 2020 - to present year

Option e: Mavis Wright, 1926 – 1975 Stephen Forster and Heather Willis, 2020 - to present year

A

Option E is the correct answer. The solicitor can rely on the existing search against Reginald Forster but must search against all the pre-root owners of whom they are aware and the personal representatives of Reginald Forster. As there is uncertainty about the exact years Mavis Wright owned the land, the solicitor should search back to 1926, when the land charges system was introduced.

Option A is wrong because the solicitor must search against all the pre-root owners of whom they are aware and the personal representatives of Reginald Forster as these are all estate owners revealed in the epitome of title.

Option B is wrong because it is not necessary to search against a lender only estate owners (unless the lender is in possession but there is nothing on the facts to suggest this is the case).

Option C is wrong because the solicitor can rely on the existing search against Reginald Forster.

Option D is wrong because the solicitor must search against all the pre-root owners of whom they are aware, namely Mavis Wright.

35
Q

A commercial property client is interested in purchasing some undeveloped land but suspects that the local people may have common rights over it. The land is registered, and contracts have not yet been exchanged.

Which of the following searches and enquiries would confirm whether the local people have common rights over the land?

A. Local Land Charges Search (LLC1).

Option b: Land Charges Search (K15).

Option c: Enquiries of the Local Authority (CON29).

Option d: Optional Enquiries of Local Authority (CON29O).

Option e: Environmental search.

A

Option D is correct. The Optional Enquiries of Local Authority contain Enquiry 22, about registered common land. It is important to raise this optional enquiry where a client is purchasing land that has not previously been developed in order to find out whether there are any common rights over it.

Option A is wrong as the Local Land Charges Search (LLC1) only reveals any financial charges or restrictions on land that have been imposed by public authorities under statute.

Option B is wrong as a Land Charges Search (K15) search is only carried out in respect of unregistered land.

Option C is wrong as Enquiries of the Local Authority (CON29) are general enquiries which are relevant to every transaction and do not cover enquiries which are more specialised in nature such as the one raised here by the client about registered common land.

Option E is wrong as an Environmental search will give details of past uses of the land which, while helpful, will not indicate whether local people have common rights over it.

36
Q

A solicitor is acting for a client who is a nurse and is buying a freehold property. The property is old and dilapidated and the client has concerns that it might have subsidence as well as damp. There are also concerns that the rear extension is not structurally sound. The client is buying with the aid of a mortgage for 40% of the value of the property.

The lender has agreed to proceed if the client obtains a ‘valuation’. The solicitor has agreed to personally inspect the property to check out a potential issue with a right of way.

The buyer has surplus money available to pay for additional costs related to the purchase.

What advice should the solicitor give the client regarding how the client should ascertain the physical condition of the property?

A. The client should rely on the lender’s ‘valuation’.

Option b: The client should commission and rely on a ‘homebuyers’ valuation and survey’.

Option c: The client should rely solely on their own personal inspection of the property.

Option d: The client should rely on the solicitor’s personal inspection of the property.

Option e: The client should commission and rely on a full structural survey.

A

Option E is the correct answer. A full structural survey will give the client the most detailed information and the most protection. Given the physical state of the property the client will need as full a picture as possible of the extent of the disrepair to better inform them of what the costs implications will be. Full structural surveys are expensive but the client seems to have surplus funds available to pay for this.

Option A is wrong as the purpose of the valuation is to assess whether the property is adequate security for the loan and will not necessarily tell the client much about the state of the structure of the property.

Option B is wrong as the ‘homebuyers’ valuation and survey’ will contain advice on necessary repairs and maintenance but will be less detailed than a structural survey.

Option C is wrong as the client is not an expert.

Option D is wrong as although the solicitor might identify some additional issues with the property, the solicitor is not an expert on the physical condition of the property.

37
Q

A developer client has acquired an office building. The office use falls within class E of the Town and Country Planning (Use Classes) Order 1987(‘the Use Classes Order’). The client proposes to remove some internal load-bearing walls from the building and change its use to retail (within class E of the Use Classes Order).

What advice should be given to the client as to whether its proposals constitute ‘development’ for the purposes of section 55 of the Town and Country Planning Act 1990?

A. Both the change of use and building works are ‘development’ because the change of use is material, and the building works are structural.

Option b: The change of use is ‘development’ because it is material, but the building works are not development because they are internal.

Option c: Neither the change of use or the building works are ‘development’ because there is no change in the class of use (under the Use Classes Order) and the building works are internal.

Option d: Neither the change of use or the building works are ‘development’ because they are both permitted development under the Town and Country Planning (General Permitted Development) Order 2015.

Option e: Both the change of use and the building works are ‘development’ because they will be carried out by a developer client.

A

Option C is correct. The definition of ‘development’ excludes works that only affect the interior of a building and changes of use that are within the same class of use under the Use Classes Order.

Option A is wrong. The definition of ‘development’ does cover both a material change of use and building works, but certain matters are excluded (as described above).

Option B is wrong, because the change of use is excluded from the definition of development due to both office and retail falling within the same Use Class (Class E).

Option D is wrong. The GPDO automatically grants planning permission for certain types of ‘development’. As neither proposal constitutes development, the GPDO is irrelevant.

Option E is wrong. It is irrelevant for the purposes of the statutory definition of ‘development’ who is carrying out the works.

38
Q

A solicitor is acting for a client who is buying a freehold property (‘the Property’). The client is buying the Property because the client likes the uninterrupted view across the field in front of the Property (‘the Field’) to the sea. The Field does not form part of the Property. The solicitor plans to submit a local search comprising CON29 enquires of the local authority, a local land charges search and CON29 optional enquires of the local authority.

Will carrying out a local search in respect of the Property reveal if any building is currently planned on the Field?

A. Yes, because a local land charges search and a CON29 search of the Property will reveal any current planning consents to build on the Field.

Option b: Yes, because a CON29O search of the Property will reveal any current planning consents to build on the Field.

Option c: No, because the local search of the Property will not reveal planning consents to build on the Field.

Option d: No, because current planning consents to build on land are not found in the results of the local search.

Option e: Yes, because a local land charges search will reveal any current planning consents to build on the Field.

A

Option C is the correct answer. The local search will only reveal details of planning consents in relation to the Property, not to adjoining property.

Option A is wrong as the local search will only reveal details of planning consents in relation to the Property, not to adjoining property.

Option B is wrong as the CON 29O does not reveal details of planning consents and in any event the local search will only reveal details of planning consents in relation to the Property, not to adjoining property.

Option D is wrong as the details of planning consents are found in the results of the local search.

Option E is wrong as the local land charges search will only reveal details of conditional planning consents in relation to the Property, not to adjoining property.

39
Q

A solicitor acting for a client on the purchase of an unregistered freehold property receives an epitome containing the following documents which do not reveal the names of any other estate owners:

A conveyance dated 1980 between Anna Colt and Harpreet Singh

A mortgage dated 1980 between Greenock Bank and Harpreet Singh which was discharged in 2005.

A death certificate of Harpreet Singh dated this year

A grant of probate granted to Maminda Singh this year.

The epitome does not contain any land charges searches.

Who should the solicitor obtain land charges searches in respect of and for what period?

A. Anna Colt from 1926 to 1980, Harpreet Singh from 1980 to the current year, Greenock Bank from 1980 to 2005 and Maminda Singh for this year.

Option b: Anna Colt from 1926 to 1980, Harpreet Singh from 1980 to the current year and Maminda Singh for this year.

Option c: Anna Colt from 11926 to 1980, Harpreet Singh from 1926 to the current year and Maminda Singh for this year.

Option d: Anna Colt from 1926 to 1980, Harpreet Singh from 1926 to the current year, Greenock Bank from 1980 to 2005 1980 and Maminda Singh for this year.

Option e: Anna Colt from 1926 to 1980 and Harpreet Singh from 1980 to the current year.

A

Option B is the correct answer. The solicitor must search against the estate owners of the property for their period of ownership of the property. It is not known when Anna Colt acquired the property and so it is necessary to search against her name back to 1926 when the registers began. We know that Harpreet Singh only owned the property from 1980 until this year when he died and so this is the period we must search for. Maminda Singh only became executor this year when all the deceased’s property would have vested in her and so we will need to search against her for this year only.

Option A is wrong. Searches will be made against Anna Colt, Harpreet Singh and Maminda Singh for the reasons above. There is no need to search against Greenock Bank as the lender is not an estate owner.

Option C is wrong. The searches against Anna Colt and Maminda Singh are correct, but there is no need to search against Harpreet Singh further back than 1980 as we know he only acquired the property then.

Option D is wrong as although the searches against Anna Colt and Maminda Singh are correct, there is no need to search against Harpreet Singh further back than 1980 as we know he only acquired the property then and there is no need to search against Greenock Bank as the lender is not an estate owner.

Option E is wrong as although the searches against Anna Colt and Harpreet Singh are correct, a search against Maminda Singh is also required.

40
Q

A solicitor acts for a client who is buying a registered freehold property (‘the property’). The property has a large tree in the front garden which the client is hoping to cut down.

Which of the following will reveal whether the tree is subject to a tree preservation order (TPO)?

A. The existence of a TPO will be revealed by the solicitor raising a Local Land Charges Search (LLC1).

Option b: The existence of a TPO will be revealed by the solicitor raising Optional Enquiries of the Local Authority (CON29O).

Option c: The existence of a TPO will be recorded as an entry in the Charges Register on the official copies of title to the property.

Option d: The existence of a TPO will be revealed by the solicitor raising a Forestry Commission report.

Option e: The existence of a TPO will be revealed by undertaking a search of the index map.
E
The existence of a TPO will be revealed by undertaking a search of the index map.

A

The correct option is A. Local land charges are financial charges or restrictions on land that have been imposed by public authorities under statute. A Local Land Charges Search on form LLC1 will provide details of any TPOs.

Option B is wrong because whilst the CON29O form allows an optional enquiry to be raised in relation to hedgerow notices, it does not give information about TPOs.

Option C is wrong because a TPO is a restriction on land that has been imposed by a public authority under statute. They are not apparent from an investigation of the title to the property.

Option D is wrong because there is no formal search or enquiry to be made to the Forestry Commission concerning TPOs.

Option E is wrong because an index map search will confirm whether the property is registered but the search does not provide detail of TPOs.

41
Q

A group of five friends bought a property together six years ago; they all worked locally and it worked out cheaper than each of them buying their own property. The property was registered in the names of the first four friends and the transfer stated that they held the property as tenants in common in equity in equal shares. Two of the friends are now a couple and want the property to be sold as quickly as possible so that they can buy a property just for themselves. In addition, a third friend also wants to sell as she has just accepted the offer of a new job in a city 100 miles away. The remaining two friends insist that the property should not be sold.

Which of the following statements best describes the process for resolving the dispute?

A. The two friends who do not want to sell the property must agree to the wishes of the majority who do want to sell.

Option b: Any of the five friends may apply to the court for an order to determine whether or not the property should be sold and the court may make such order as it thinks fit.

Option c: If an application to the court for an order to determine the dispute between the friends is made, the court must give the greatest weight to the purposes for which the property subject to the trust is held.

Option d: Any of the five friends may apply to the court for an order to determine whether or not the property should be sold, but the court cannot determine the application without the consent of all of the registered owners.

Option e: In determining any application for an order to resolve the dispute between the friends, the court does not have to consider the wishes of anyone who is not a trustee of the property.

A

Option B is correct. Any person who is a trustee or has an interest in the property subject to the trust may make an application to the court and the court may make such order as it thinks fit (s14 Trusts of Land and Appointment of Trustees Act 1996).

Option A is wrong. The two friends in the minority do not have to agree with those with the majority opinion.

Option C is wrong. The court must have regard to the various factors in s15, but nothing in the 1996 Act dictates that the purpose or use of the property must be given greater weight than other factor. If any factor is to be given greater weight, it is the interest of any secured creditor or mortgagee.

Options D and E are wrong. The application does not require the consent of the trustees and the court should consider the wishes of the beneficiaries.

42
Q

A solicitor is acting on the purchase of a freehold property that is used for commercial offices. The property is not situated close to a river or the coast. The solicitor has carried out a Local Land Charges Search (LLC1), has raised Enquiries of the Local Authority (CON29) and has raised water and drainage enquiries of the local statutory undertaker. In addition, the solicitor has raised pre-contract enquiries with the seller.

Should the solicitor also consider carrying out a flood search as part of the pre-contract searches and enquiries on the property?

A. Yes, because the property may be at risk from surface water flooding and this will be revealed by a flood search.

Option b: Yes, because a solicitor must carry out a flood search for every property where acting for a buyer.

Option c: No, because insurance against flood risk is guaranteed to be available under the Flood Re scheme, so there is no need to carry out a flood search.

Option d: No, because the main risk of flooding is from nearby water courses and this property is too far away to be affected.

Option e: No, because the solicitors acting for the seller of the property will reveal details of any flooding in answer to pre-contract enquiries.

A

Option A is the correct answer as damage can come from surface water flooding even if the property is not close to a water course or the coast.

Option B is wrong since it is not mandatory to carry out any search or enquiry however failure by the buyer’s solicitor to make searches may give rise to liability in negligence.

Option C is wrong as the insurance under the Flood Re scheme only applies to residential properties.

Option D is wrong as surface water flooding is a risk that should be investigated for every property.

Option E is wrong as the information provided by the seller may be incomplete and the seller has only a very limited duty of disclosure.

43
Q

A solicitor is acting for a client who is purchasing some offices for their new business. The property was built in the 19th century and was originally used as a large family home. It was converted into offices in 1995 and appears to be in need of minor repair. The client intends to extend the property so as to add a reception area. The client is funding half the purchase price of the property from savings and the remainder will be funded by way of mortgage. Contracts have not yet been exchanged. The client seeks the solicitor’s advice as to whether they should carry out a survey on the property and, if so, the type of survey they would recommend.

Which of the following statements best describes the advice the solicitor should give?

A. It is unnecessary to carry out a survey and given how expensive surveys are they should only be carried out where a property is obviously in significant disrepair.

Option b: The lender will only require the client to pay for a valuation of the property. This is the cheapest option and is adequate for the client’s purposes.

Option c: The client should obtain a Homebuyers’ Valuation and Survey as this will give advice as to any necessary repairs and maintenance requirements.

Option d: The client could consider instructing a surveyor to carry out a full structural survey; however, this is expensive and is only really needed when a property is in need of substantial repair work.

Option e: The client should instruct a surveyor to carry out a full structural survey as the property is in need of some repair and the client intends to carry out further alterations.

A

Option E is the best answer because a full structural survey is always advisable where the property in question is in disrepair or where further alterations are proposed – both of which are relevant on these facts. Additionally, the age of the property and the fact it has previously been converted would also point to the need for a structural survey.

Option A is wrong because the property is in disrepair and further alterations are proposed and either of these factors would suffice on its own to lead to a recommendation of a full structural survey.

Option B is not the best answer because although this may be all that the lender will require, it will only assess whether the property is adequate security for the loan (which on these facts is for only half the value of the property). It will not necessarily tell the client much about the structural integrity of the property.

Option C is not the best answer because although this type of survey is more detailed than a simple valuation, it is still less detailed than a structural survey and is only really relevant to residential properties (as the name of the survey suggests).

Option D is not the best answer because although a full structural survey is the most expensive option, on these facts it would seem to be commercially advisable.

44
Q

A solicitor is acting for a buyer in a conveyancing transaction and intends to exchange contracts with the seller’s solicitor using Law Society formula B later today. The contract incorporates the Standard Commercial Property Conditions (Third Edition) (SCPC). The buyer’s solicitor is holding cleared funds for the deposit of 10% and intends to send the deposit to the seller’s solicitor immediately after exchange by the method stipulated in the SCPC. However, the buyer’s solicitor cannot send their client’s part of the contract to the seller’s solicitor until tomorrow.

Which of the following best describes whether variations will be required to formula B on exchange?

A. No variations are required to formula B.

Option b: Two variations are required to allow the buyer’s solicitor to send the deposit via banker’s draft or cheque and not via electronic means and to put their client’s part of the contract in the first-class post or document exchange (DX) tomorrow not today.

Option c: The only variation required is to allow the buyer’s solicitor to send the deposit via electronic means and not via banker’s draft or cheque.

Option d: The only variation required is to allow the buyer’s solicitor to put their client’s part of the contract in the first class post or DX tomorrow not today.

Option e: Two variations are required to allow the buyer’s solicitor to send the deposit via electronic means and not via banker’s draft or cheque and to put their client’s part of the contract in the first class post or DX tomorrow not today.

A

Option E is the correct answer.

Two variations to formula B are required in this transaction so options A, C and D are wrong.

Firstly, the deposit will be sent by electronic means as per Standard Commercial Property Condition 3.2.2 rather than by banker’s draft or cheque as provided for in the undertakings contained in formula B.

This means option B is also wrong. Secondly, the Buyer’s solicitor will not be able to comply with the undertaking in formula B to put their client’s part contract in the first class post/DX today so will need to vary the formula to provide that this will be done tomorrow instead.

45
Q

A man owns a house with a large garden. In the garden there is a greenhouse which was built in 1985. The man has been growing vegetables in the greenhouse since he moved in to the house 25 years ago. A woman bought the freehold of the bungalow next door to the man’s property and just over 12 months ago planted some fast growing fir trees along the boundary of the two properties. The man has never been given permission to enjoy uninterrupted light to the greenhouse and he is now worried that the trees will soon block the light to his greenhouse.

Does the man have a right to light to the greenhouse arising by prescription?

A. Yes, because the right has been used continuously for at least 20 years without force, secrecy or permission and so the easement is automatically created under statute.

Option b: Yes, because the right has been used continuously for at least 20 years without force, secrecy or permission and so the easement will be created under the common law as it will be presumed that it has been in existence since 1189.

Option c: Yes, because the right has been used continuously for at least 20 years without force, secrecy or permission and an easement will therefore be deemed to exist under the doctrine of lost modern grant.

Option d: No, because it is not possible to establish a right to light to the greenhouse as the right is too wide or vague and therefore is not capable of being an easement.

Option e: No, because a right to light can only be established under statute and as the woman planted the trees over 12 months ago, the man has now lost the right to apply for an easement under statute.

A

Option C is correct. The doctrine of lost modern grant will operate to automatically create an easement as soon as 20 years continuous use has been achieved, providing the common law conditions are met. The easement was therefore created, at the latest, 5 years ago.

Option A is wrong as it is necessary to make an application to the court to establish an easement under the Prescription Act 1832. Easements are not created automatically under the Act.

Option B is wrong as the presumption at common law of use since 1189 is easily rebutted. Here, the greenhouse was only built in 1985 so the right to light cannot have been enjoyed since 1189.

Option D is wrong as whilst a general right to light would be too wide or vague to satisfy the requirements for a valid easement, it is possible to claim a right to light through a defined aperture, such as through a greenhouse.

Option E is wrong as the Prescription Act is not the only method available to establish a right to light. If the Act is used, an interruption in use of more than 12 months could be fatal to the application. However, the trees were only planted just over 12 months ago and they are not, as yet, blocking the light to the greenhouse.

46
Q

A solicitor is advising the purchaser of a commercial property. The contract for the sale of the property has been drafted by the seller’s solicitor and sent to the purchaser’s solicitor for review. The draft contract incorporates the Standard Commercial Property Conditions (Third Edition – 2018 Revision) (‘SCPC’).

The parties agreed in the heads of terms that the deposit amount would be 10% of the purchase price, payable on exchange of contracts, to be held by the seller’s solicitor as stakeholder.

The parties have also agreed to exchange using The Law Society telephone/telex exchange – Formula B (‘Formula B’). The seller’s solicitor has confirmed a willingness to make variations to Formula B if required.

Does the purchaser’s solicitor need to draft any special conditions for the contract, to deal with the payment of the deposit?

A. Yes, because it would be preferable for the purchaser if the deposit was held by the seller’s solicitor as agent.

Option b: Yes, because otherwise the deposit will be payable ‘by electronic means’ which would preclude the use of Formula B for exchange.

Option c: Yes, because expressly stating matters in a special condition is necessary in order to bring matters to the parties’ attention prior to exchange.

Option d: No, because the SCPC accurately reflect the position as agreed between the parties.

Option e: No, because the SCPC provides for a deposit of 5% of the purchase price which is more favourable for the purchaser.

A

Option D is correct. Condition 3.2 of the SCPC states that the buyer is to pay a deposit of 10% of the purchase price and that this is to be paid by electronic means and will be held by the seller’s conveyancer as stakeholder. The unamended SCPC therefore reflect the heads of terms.

Option A is wrong, as it is preferable for the buyer if the deposit is held as stakeholder (not agent) and indeed, this is what has been agreed in the heads of terms.

Formula B can be varied at the time of exchange to make it clear that the deposit is being sent by electronic means (rather than by client account cheque or banker’s draft as Formula B specifies). As a result, the SCPC do not need amending to deal with this, as Formula B can still be used, albeit with a variation. Option B is not, therefore, the best answer.

A special condition may be used to deal with the deposit expressly (in a firm’s precedent contract for example). However, where the SCPC reflect the heads of terms, this is not necessary and Option C is not, therefore, the best answer.

The SCPC provide for a 10% deposit and therefore Option E is wrong.

47
Q

A solicitor is acting for a purchaser of a commercial property which is being partly funded by a 75% mortgage. The solicitor has undertaken all the relevant pre-contract searches and enquiries and has reported on title to their client who is happy to proceed to exchange. There are no legal issues with the property.

The solicitor has also provided a draft Certificate of Title to the lender.

Which of the following best describes the most likely form and content of the Certificate of Title in this transaction?

A. The Certificate will confirm that the property has a “good and marketable title”. The solicitor will have used a form approved by the Law Society and UK Finance.

Option b: The Certificate will confirm that the property has a “good and marketable title”. The solicitor will have used the City of London Law Society Certificate of Title.

Option c: The Certificate will contain a summary of the pre-contract searches and enquiries and will confirm that the property has a “good and marketable title”. The solicitor will have used a form approved by the Law Society and UK Finance.

Option d: The Certificate will contain a summary of the pre-contract searches and enquiries and will confirm that (subject to any Disclosures made in the Certificate) the property has a “good and marketable title”. The solicitor will have used the City of London Society Certificate of Title.

Option e: The Certificate will contain a summary of the pre-contract searches and enquiries and will confirm that the property has a “good and marketable title”. The solicitor will have used their own precedent.

A

Option D is correct because this is a commercial transaction and the lender will require a more detailed Certificate of Title than that which is required in residential transactions. As it is a commercial transaction, the solicitor is likely to have used the City of London Society Certificate of Title.

Option A is wrong because the form approved by the Law Society and UK Finance is more appropriate for residential transactions.

Option B is wrong because the Certificate of Title in a commercial transaction will contain more detail than just confirming the property has a “good and marketable title”.

Option C is wrong because although the Certificate of Title will contain the level of detail, it would not be in the form approved by the Law Society and UK Finance as that is more appropriate for residential transactions.

Option E is wrong because it is unlikely that the solicitor would use their own precedent.

48
Q

A solicitor is acting for two clients in the joint purchase of an unregistered freehold property. It is a cash purchase. The seller of the property purchased it 50 years ago from a builder. Contracts were exchanged 21 days ago. The seller’s solicitors supplied land charges search results with the epitome prior to exchange, the results of which were all clear. The buyer’s solicitor has also carried out a pre contract land charges search against the seller, the results which were also clear.

Which of the following searches should the buyer’s solicitor undertake between exchange and prior to completion?

A. Land charges search (K15) against the seller for the whole period of their ownership

B. Land charges search (K16) against the seller

C. Land Registry search with priority of the whole of the land in the registered title (OS1)

D. Land charges search (K15) against the seller for the last 15 years of their ownership

E. Land charges searches (K15) against the seller for the whole period of their ownership and the builder from 1926 to the year they sold to the seller

A

Option A is the correct answer. A land charges search against the seller has already been carried as a pre-exchange search but will need to be repeated against the current seller’s name (unless the transaction can be completed within the 15 working day priority period conferred by the pre-exchange search-this is not the case here).

Option B is wrong as the Land charges search (K16) is a bankruptcy only search and will not reveal all possible entries against the seller’s name.

Option C is wrong as the Land Registry search is a registered land search and does not apply to unregistered land

Option D is wrong. The land charges search (K15), should be against the seller for the period that they owned the land which here is for longer than 15 years.

Option E is wrong. Whilst it would be correct to search against the seller for the whole of the period of their ownership it is not necessary to search against the names of the previous estate owners (the builder) as a land charges search against the builder would have either been included in the Epitome of Title sent by the seller’s solicitors or been carried out prior to exchange by the buyer’s solicitors and no entries can be registered against them after they parted with the property.

49
Q

You are acting for the purchaser of a registered freehold property which is subject to a mortgage with Barclay’s Bank. Having exchanged contracts for completion set for two weeks from now, you receive the completion information and undertakings form (TA13) from the seller’s solicitors.

What information would you normally expect to be revealed by the seller’s solicitors in this form?

A. Details of the amount outstanding to Barclay’s Bank, an undertaking given by the seller’s solicitors to redeem the mortgage with Barclay’s Bank and confirmation of the exact amount required to complete the purchase.

B. Confirmation of the amount of council tax due on the property, an undertaking given by the seller’s solicitors to redeem the mortgage with Barclay’s Bank and confirmation of the exact amount required to complete the purchase.

C. Details of the whereabouts of the keys to the property, an undertaking given by the seller’s solicitors to redeem the mortgage with Barclay’s Bank and confirmation of the exact amount required to complete the purchase.

D. Information about what additional items are included in the sale and what is not included, an undertaking given by the seller’s solicitors to redeem the mortgage with Barclay’s Bank and confirmation of the exact amount required to complete the purchase.

E. Details about who is responsible for the repair of the boundaries at the property, an undertaking given by the seller’s solicitors to redeem the mortgage with Barclay’s Bank and confirmation of the exact amount required to complete the purchase.

A

Option C is the correct answer. This information is all contained in the Completion Information and Requisitions form (TA13).

Option A is wrong. Whilst it is correct that the form will contain an undertaking given by the seller’s solicitors to redeem the mortgage with Barclay’s Bank and confirmation of the exact amount required to complete the purchase there would not be any detail about the amount required to redeem that mortgage. This is not needed as the buyer’s can rely on the undertaking given by the seller’s solicitors.

Option B is wrong. Whilst it is correct that the form will contain an undertaking given by the seller’s solicitors to redeem the mortgage with Barclay’s Bank and confirmation of the exact amount required to complete the purchase there would not be any detail about the amount of council tax due on the property. This information should be revealed prior to exchange in the pre contract enquiries of the seller.

Option D is wrong. Whilst it is correct that the form will contain an undertaking given by the seller’s solicitors to redeem the mortgage with Barclay’s Bank and confirmation of the exact amount required to complete the purchase, there would not be any detail about what additional items are included in the sale and what is not included.

Option E is wrong. Whilst it is correct that the form will contain an undertaking given by the seller’s solicitors to redeem the mortgage with Barclay’s Bank and confirmation of the exact amount required to complete the purchase, there would not be any detail about who is responsible for the repair of the boundaries at the property. This information should be revealed prior to exchange in the pre contract enquiries of the seller.

50
Q

A solicitor acts for a corporate client who is taking an assignment of a lease granted in 2017 for 30 years. Contracts have been exchanged and completion is scheduled to take place in two weeks’ time. Title to the lease is registered at HM Land Registry.

Which of the following statements best describes the requirement to register the assignment after completion?

A. The solicitor should register the client as proprietor of the lease within 14 days of completion of the assignment.

B. The solicitor should register the client as proprietor of the lease within 21 days of completion of the assignment.

C. The solicitor should register the client as proprietor of the Lease within the priority period stated in the OS1R.
selected

D. The solicitor should register the client as proprietor of the Lease within two months of completion of the assignment.

E. No application to register is required because the client will become the legal proprietor of the lease on the date of completion of the assignment.

A

Option C is correct. Prior to completion of the assignment, the solicitor should carry out an OS1 search at Land Registry. Carrying out that search generates a priority period and the solicitor should register the client as proprietor of the lease within the priority period stated in the OS1R. The priority period provides protection against any entries which may be placed on the register after the date of the search but before the assignee is registered as proprietor of the lease.

Option A is wrong. 14 days after completion is the deadline for the payment of any Stamp Duty Land Tax that is due.

Option B is wrong. 21 days after completion is the deadline for registration at Companies House of a charge (i.e. mortgage) created by a company.

Option D is wrong. Two months after completion is the deadline for first registration of a transaction involving unregistered land which triggers the requirement to register.

Option E is wrong as the client will only become registered proprietor of the lease at the date of registration, not at the date of completion of the assignment. An application to register is therefore essential.

51
Q

A client is the buyer of a registered freehold property. Both the client and the seller are companies.

Contracts have been exchanged and the buyer’s solicitor is preparing for completion. The buyer is not taking out a mortgage.

Which of the following statements best describes the searches that will be carried out before completion?

A. The buyer’s solicitor will need to carry out an official search of whole at Land Registry (‘OS1’) and a company search against the buyer.
selected

B. The buyer’s solicitor will need to carry out an OS1 and a bankruptcy search against the seller.

C. The only pre-completion search the buyer’s solicitor will need to carry out is an OS1.

D. The reason the buyer’s solicitor will do an OS1 is to reveal any insolvency proceedings concerning the seller.

E. The buyer’s solicitor will need to carry out an OS1 and a company search against the seller.

A

The buyer’s solicitor will need to carry out an OS1 search to check whether there have been any changes to the title and to provide a priority period. A company search should also be carried out against the seller before the completion monies are handed over. This is to check the seller is not insolvent. So, Option E is the best answer.

As there is no mortgage, there will be no need to carry out a company search against the buyer, so Option A is not the best answer.

A bankruptcy search is only made against an individual and so Option B is not the best answer.

As stated above, a company search is required to establish whether there are any insolvency proceedings against the seller and so Option C is not the best answer.

Insolvency proceedings relating to a company do not appear on the title register at Land Registry and so Option D is wrong.

52
Q

The seller’s solicitor and the buyer’s solicitor are intending to adopt the Law Society’s Code for Completion by Post.

Which one of the following statements is correct?

A. The seller’s solicitor can make a reasonable charge for acting as the buyer’ solicitor’s agent for the purposes of completion.

B. The buyer’s solicitor will know that completion has taken place before they receive the relevant documentation through the post or the document exchange.

C. If the seller’s solicitor fails to carry out the buyer’s solicitor’s instructions, the only sanction against the seller’s solicitor is that they can be disciplined for professional misconduct.

D. The buyer’s solicitor should give their instructions to the seller’s solicitor by telephone and make a contemporaneous note that they have done so.

E. The buyer’s solicitor must ensure that the buyer is aware that completion is to take place by post.

A

Option B is the correct answer because the Law Society’s Code for Completion by Post (“the Code”) requires the seller’s solicitor to inform the buyer’s solicitor immediately that completion has taken place.

Option A is wrong because the Code provides that the seller’s solicitor will act as the buyer’s solicitor’s agent without a fee.

Option C is wrong because the seller’s solicitor would be liable through the court.

Option D is wrong because the buyer’s solicitor should instruct the seller’s solicitor in writing.

Option E is wrong because the decision to adopt the Code is a matter for the buyer’s solicitor and the seller’s solicitor and there is no requirement under the Code or any other obligation on the buyer’s solicitor to inform the buyer.

53
Q

A solicitor acts for a married couple who are the buyers of a registered freehold property. The proprietorship register reveals the existence of a chain of indemnity covenants in respect of restrictive and positive covenants in the charges register.

Do the seller and the buyers all need to sign the transfer deed?

A. No, because only the seller is required to sign the transfer on every transaction and the buyers can sign if they wish.

B. Yes, because the seller and the buyers must all sign the transfer on every transaction.

C. Yes, because the seller always signs and the buyers will make a declaration of trust and give an indemnity covenant.

D. Yes, because seller always signs and the buyers will make a declaration of trust.

E. Yes, because the seller always signs and the buyers will give an indemnity covenant.

A

The seller always needs to sign the transfer, but the buyer only needs to sign in certain situations. The relevant reasons why the buyers need to sign on this occasion is that they will make a declaration of trust (as they will be co-owners) and give an indemnity covenant in the transfer (to continue the chain of indemnities). So, Option C is correct.

Option A is wrong, as the buyers do not have a choice on these facts.

Option B is wrong, as a buyer only needs to sign in certain situations.

Options D and E are wrong, as they do not identify the 2 reasons that necessitate the buyers’ signatures.

54
Q

A company has just bought a registered land commercial property. The purchase price was £1,5000,000 and the company bought with the aid of a mortgage. The lender’s solicitor is dealing with the payment of stamp duty land tax (‘SDLT’) and any post-completion applications.

What steps should the lender’s solicitor take?

A. The solicitor should pay SDLT within 14 days, register the mortgage within 21 days and make applications to Land Registry within the priority period.

B. The solicitor should pay SDLT within 14 days, register the mortgage within 28 days and make applications to Land Registry within the priority period.

C. The solicitor should pay SDLT within 14 days, register the mortgage with 21 days and make applications to Land Registry within two months.

D. The solicitor should pay SDLT within 30 days and make applications to Land Registry within the priority period.

E. The solicitor should pay SDLT within 30 days, register the mortgage with 21 days and make applications to Land Registry within the priority period.

A

Option A is the correct answer. The purchase price means SDLT is due and the deadline for payment (to avoid penalties) is 14 days from completion. The buyer, a company, has entered into a mortgage and that mortgage must be registered at Companies House within 21 days of completion (or the mortgage is void against creditors). The solicitor should apply to register the transfer to the client and the new mortgage at Land Registry. This should be done within the priority period conferred by the pre-completion official search of whole (OS1), to ensure the applications have priority over any others lodged with Land Registry.

Option B is wrong. The mortgage must be registered within 21 days.

Option C is wrong. The applications to Land Registry should be made within the priority period conferred by the pre-completion official search of whole (OS1), which is 30 working days from the search result date.

Option D is wrong. The SDLT should be paid within 14 days. Option D also omits registration of the mortgage at Companies House.

Option E is wrong. The SDLT should be paid within 14 days.

55
Q

A solicitor is acting for two clients (the buyers) in the joint purchase of a property, the title of which is an unregistered freehold, from a sole owner (the seller). The buyers are purchasing with the aid of a mortgage. The solicitor is also instructed to act for the lender in the transaction.

Which of the following searches should the solicitor undertake prior to completion?

A Land Registry search with priority of part of the land in a registered title (OS2) and a Bankruptcy search (K16) against the buyers

B. A Bankruptcy search (K16) against the seller and a land charges search (K15) against the buyers

C. A Land Registry search with priority of the whole of the land in a registered title (OS1) and a Bankruptcy search (K16) against the seller.

D. A Land Registry search with priority of the whole of the land in a registered title (OS1) and a Bankruptcy search (K16) against the buyers.

E. Land charges search (K15) against both the seller and the buyers.

A

Correct answer - Option E – As the title of the property is unregistered a land charges search (Form K15) is required against both the seller and the buyers. The latter to protect the lender as the buyers are purchasing with a mortgage.

Wrong answer - Option A- Whilst a bankruptcy search against the buyers should be carried out here; the property is unregistered, and the transaction is a purchase of the whole not of part so a Land Registry search with priority of part would not be appropriate.

Wrong answer - Option B - Whilst a bankruptcy search would be required to protect the lender it should be carried out against the buyers not the seller. A K15 against the buyers would reveal bankruptcy entries and this would be effective in protecting the lender.

Wrong answer - Option C - Whilst a bankruptcy search would be required to protect the lender it should be carried out against the buyers not the seller, the property is unregistered so a Land Registry search would not be appropriate.

Wrong answer - Option D - Whilst a bankruptcy search against the buyers would be required to protect the lender the property is unregistered so a Land Registry search (OS1) would not be appropriate.

56
Q

A solicitor is acting for a lender to an individual borrower purchasing a property with registered title. The lender requires a clear bankruptcy search against the name of the borrower before releasing the mortgage funds.

Which of the following statements best describes the bankruptcy search the solicitor will instruct and the priority period protection afforded by that bankruptcy search result?

A. The solicitor should instruct a K15 search against the borrower’s name and the result will afford a priority period of 15 days in which to complete the transaction.

B. The solicitor should instruct a K15 search against the borrower’s name and the result will afford a priority period of 15 working days in which to complete the transaction.

C. The solicitor should instruct a K16 search against the borrower’s name and the result will afford a priority period of 15 days in which to complete the transaction.

D. The solicitor should instruct a K16 search against the borrower’s name and the result will afford a priority period of 15 working days in which to complete the transaction.

E. The solicitor should instruct a K16 search against the borrower’s name but the result will not afford a priority period in which to complete the transaction.

A

The correct option is E. A K16 bankruptcy search does not afford a priority period in which to complete the transaction and the search should be undertaken as close as possible to the date of completion.

Options C and D are wrong in suggesting a priority period for a K16 search.

Options A and B are wrong in suggesting a K15 search against a purchaser/borrower. A K15 search would be an appropriate search against a seller in unregistered land and would reveal any entries relating to bankruptcy in relation to the seller. A K15 search result does provide a 15 working day priority period in which to complete the transaction. Completion must take place during the priority period in order for the purchaser to take free from any entries registered against the seller at the Land Charges Department during that period.

57
Q

The property market is buoyant. A client buys property to refurbish then let out or sell. The client owns the freehold of a small shop premises (the shop) which he has contracted to sell. The buyer paid a 5% deposit to be held as stakeholder. The contract incorporates the Standard Commercial Property Conditions (Third Edition) (SCPCs) according to which completion is due at 14:00 hrs today. Today, at 14:00 hrs, the buyer’s solicitor calls to say the buyer has insufficient funds to complete today and is unsure how long it will take to get them. The client needs part of the proceeds of sale quickly, to pay for refurbishing other premises in order to rent them out for profit as soon as they are refurbished. The client stands to lose money for every day that the completion is delayed and the losses are likely to exceed the amount of the deposit.

A. Serve a notice to complete immediately. If the buyer does not comply with the notice then forfeit the deposit, re-sell the shop and claim damages.

B. Claim compensation for delay under the SCPCs and immediately rescind the contract so that the deposit can be forfeited.

C. Apply to the court for an order for specific performance of the contract.
Option d: Use the deposit monies immediately and claim compensation for delay under the SCPCs.

D. Use the deposit monies immediately and claim compensation for delay under the SCPCs.

E. Claim compensation for delay under the SCPCs and then sue the buyer for breach of contract to recover any additional losses.

A

Option A is the best answer as it enables the client to obtain more money (the deposit), sooner, in order to spend on the refurbishment. As the property market is buoyant there is not a great risk that the client will not be able to find another buyer.

The amount of compensation payable under the SCPC would probably be insufficient to pay for the refurbishment and the seller cannot rescind the contract and retain the deposit without serving a notice to complete. Option B is therefore wrong.

Orders for specific performance are rarely made by the court in these circumstances and would involve potentially lengthy court proceedings. So, C is not the best answer.

The deposit monies cannot be given to the client to use without first serving a notice to complete and waiting for the time for compliance to expire. So, Option D is wrong.

Option E is not the best answer as it requires a court action to claim the additional losses and so will not provide funds to the client quickly enough.

58
Q

A client has exchanged contracts to buy a freehold commercial property and the Standard Commercial Property Conditions (Third Edition-2018 Revision) (SCPCs) were incorporated into the contract. Completion is due to take place tomorrow. Unfortunately, due to some unrelated matters the client is concerned that they will not be ready to complete tomorrow and will have to delay for a few days. The client is unsure of the implications if they have to delay the purchase.

Which of the following best describes the immediate implications for the client?

A. As soon as the contractual completion date has passed, the seller can serve a notice to complete making time of the essence.

B. The seller can forfeit the deposit as soon as the contractual completion date has passed.

C. The client will not have to pay any compensation under the Standard Commercial Property Conditions.

D. As soon as the contractual completion date has passed, the seller can rescind the contract.

E. The seller can make a claim for damages for breach of contract immediately after the contractual completion date.

A

Option A is correct. As soon as the contractual completion date has passed the seller could serve a notice to complete making time of the essence.

Option B is wrong. The seller cannot forfeit the deposit as soon as the contractual completion date has passed. The seller would first need to serve a notice to complete making time of the essence and only if completion does not take place on the new completion date would they be able to forfeit the deposit.

Option C is also wrong. The client will have to pay compensation under the SCPCs.

Option D is also wrong. The delay could result in the contract being rescinded but this would only happen under the SCPCs if a notice to complete is served by the seller and then the client does not complete by the new completion date.

Option E is not the best answer. The seller might be able to make a claim for damages for breach of contract but the seller will have to wait to see what its losses are before deciding to sue. The seller may have no loss to sue for if they are covered by contractual compensation and forfeiture of the deposit or, indeed, if the buyer does complete with only minimal delay.

59
Q

The seller is a small housing developer. The seller has agreed to sell one of the plots on the development. The title to the development is registered. The seller’s solicitor is putting in place arrangements for the release of part of the land from the registered charge in time for completion.

Which statement most accurately describes the correct position?

A. The solicitor will ensure that the mortgagee executes a vacating receipt on the mortgage deed.

B. The solicitor will check that the mortgagee is going to use the electronic notifications of discharge method (ENDs) to discharge the registered charge.

C. An electronic discharge of the registered charge (e-DS1) will be sent by the mortgagee’s computer system direct to the Land Registry.
selected

D. The seller’s solicitor will prepare form DS3 for the mortgagee to release the plot from the registered charge.

E. The seller’s solicitor will prepare form DS1 for the mortgagee to release the plot from the registered charge.

A

Option D is the correct option because form DS3 is used to release part of the land that is subject to a registered charge.

Option A is wrong because vacating receipts are used to discharge mortgages in the unregistered title system and title in this question is registered.

Option B is wrong because the electronic notifications of discharge method (ENDs) to discharge the registered charge was discontinued in January 2010.

Option C is wrong because an electronic discharge of the registered charge (e-DS1) can only be sent to discharge the whole mortgage.

Option E is also wrong because the form DS1 is used to cancel the registered charge and the mortgagee will require security over the remaining part of the land.

60
Q

A solicitor is acting for a husband and wife who are buying a residential property. The buyers’ solicitor is preparing the transfer deed and considering who needs to execute it. The buyers are buying the whole of the registered title, will hold the property as tenants in common and will not be entering into an indemnity covenant.

Should both the seller and buyers execute the transfer deed?

A. No, because whilst the seller always executes the transfer deed, the buyers would only execute if they were entering an indemnity covenant.
Option b: No, because whilst the seller always executes the transfer deed, the buyers would only execute if the sale involved a transfer of part with the creation of new covenants.

B. No, because whilst the seller always executes the transfer deed, the buyers would only execute if the sale involved a transfer of part with the creation of new covenants.

C. No, because none of the parties are required to execute the transfer deed, although they can do so if they wish.
Option d: Yes, because all parties must always execute the transfer deed.

D. Yes, because all parties must always execute the transfer deed.

E. Yes, because the seller always executes and the buyers will be declaring in the transfer deed how they own the beneficial interest.

A

Option E is the correct answer. The seller will always execute the transfer deed to transfer the land. The buyer or buyers do not always have to execute but will need to in certain circumstances. These circumstances include where they make a declaration. The facts indicate the buyers are co-owners who will hold the property as tenants in common and they will need to make a declaration to this effect in the transfer deed. As a result of this declaration, they will need to execute the transfer deed.

Option A is wrong. The buyers would need to execute if there were an indemnity covenant, but this is not the only circumstance in which they need to execute. They need to execute on these facts, because they will be declaring in the transfer deed how they hold the beneficial interest in the property.

Option B is wrong. The buyers would execute if there were a transfer of part with new covenants, but this is not the only circumstance in which they must execute. They need to execute on these facts, because they will be declaring in the transfer deed how they hold the beneficial interest in the property.

Option C is wrong. The seller must always execute and the buyer must execute because they will be declaring in the transfer deed how they hold the beneficial interest in the property.

Option D is wrong. The seller must always execute. The buyers are only required to execute in certain circumstances (such as if they enter into a covenant or make a declaration in the transfer deed).

61
Q

A solicitor is acting for the seller of a property on the discharge of the mortgage on that property. The solicitor has transferred sufficient funds to the lender to redeem the mortgage.

Which of the following statements best describes what the lender should do next?

A. The lender will complete Land Registry Form DS1 and send it through the Land Registry portal for onward transmission to the buyer’s solicitor.

B. The lender will complete Land Registry Form DS1 and send it to the seller’s solicitor for onward transmission to the buyer’s solicitor.

C. The lender will complete Land Registry Form e-DS1 and send it to the seller’s solicitor for onward transmission to the buyer’s solicitor.

D. The lender will complete Land Registry Form e-DS1 and send it to the buyer’s solicitor for onward transmission to the seller’s solicitor.

E. The lender will complete Land Registry Form OS1 and send it to the seller’s solicitor for onward transmission to the buyer’s solicitor.

A

Option B is correct for the discharge of the mortgage (Chapter 5 of the Manual). Option A is wrong because the lender would not use the portal. Option C is wrong because the lender would not send the e-DS1 to the seller’s solicitor. Option D is wrong because the lender would not send the e-DS1 to the buyer’s solicitor. Option E is wrong because you do not use the form OS1.

62
Q

A company has just completed its purchase of a registered commercial freehold property situated in England. The purchase price was £2,500,000 plus Value Added Tax (VAT). A deposit of 10% was paid on exchange of contracts and the balance was paid on completion.

Which of the statements set out below in relation to the Stamp Duty Land Tax (SDLT) payable by the company is correct?

A. The company will have to pay SDLT on the purchase price excluding VAT within 14 days of completion.

B. The company will have to pay SDLT on the amount paid on completion plus VAT within 14 days of completion.

C. The company will have to pay SDLT on the purchase price plus VAT within 30 days of completion.

D. The company will have to pay SDLT on the purchase price plus VAT before the expiry of the Land Registry priority period.

E. The company will have to pay SDLT on the purchase price plus VAT within 14 days of completion.

A

Option E is the correct answer. SDLT is payable on the total purchase price plus VAT and the deadline is 14 days from completion.

Option A is wrong because SDLT is payable on VAT.

Option B is wrong because SDLT is payable on the total purchase price inclusive of VAT.

Option C is wrong because the deadline is 14 days from completion, not 30 days.

Option D is wrong because the deadline is 14 days from completion. The Land Registry application for registration has to be submitted before the expiry of the priority period.

63
Q

A solicitor has completed the purchase of a freehold property in England on behalf of a company. The company has purchased the property with the aid of a mortgage. The solicitor is preparing to apply to the Land Registry for the registration of the company as the new registered proprietor of the property.

Which of the following documents should be submitted to the Land Registry with the Land Registry Form AP1?

A. The Land Registry OS1 search result.

B. A tax return using form SDLT1.
Option c: The original mortgage deed.

C. The original mortgage deed.

D. A certified copy of the transfer.
selected

E. The City of London Law Society (CLLS) Certificate of Title.

A

Option D is correct as this is the only document that will be submitted to the Land Registry on completion.

Option A is wrong as the search result is issued by the Land Registry and the priority period given by the search will apply automatically.

Option B is wrong as this form must be submitted to HMRC, not to the Land Registry.

Option C is wrong as only a certified copy of the mortgage deed is required.

Option E is wrong as the CLLS certificate would be submitted to the lender, not to the Land Registry.